Sie sind auf Seite 1von 123

WHATS INSIDE?

Mathematics: Algebra, Trigonometry, Statistics, Analytic


Geometry, Differential Calculus and Integral Calculus
20 items and Quick Tips

Mechanics, Thermodynamics, and Engineering Economics 20


36 items

Differential Equations and Advanced Math


20 items and Quick Tips

35

Vector Analysis and Electromagnetics


20 items and Quick Tips

51

Electrical Circuits and DC Machines


20 items

64

Analog Electronics
36 items

72

Digital Electronics
40 items

82

Industrial Electronics
30 items

92

Computer Networks and Data Communications: Part 1


40 items

97

Wave Propagation
20 items

105

Analog Communications and Broadcast Engineering


20 items

109

Digital and Data Communications: Part 2


40 items

114

GUIDELINES
The answer that cant be a solution is simply done in a complete sentence.
*Solutions are gathered in one particular section separately from the
chapter test with multiple choices. Rare solutions are shown in units. Brief
explanations are inserted to give you an information as possible. However,
there are some lacking algorithms of the mathematical solution; so it is up
to you if you would expand it until the final answer is attained. Anyway, the
calculator will compute for that case.
Please read Quick Tips where you can learn to manipulate.

SAFETY
To be honest, this latest document was created illegally. Please keep away
from those teachers who made actually the mock exam. The editor did not
like to put reasons how it is successfully compiled. We beg not to take at
risk, students. (c) Dec 2015

Mathematics

A character after each problem is indicated on what major mathematics belongs with:
A Algebra


T Trigonometry

SP
Statistics and Probability
AG
Analytic Geometry


DC
Differential Calculus
IC
Integral Calculus

1
2
3
4
5
6
7

Pretest and Post-test Items

In a pile of logs, each layer contains one more log than the layer above and the top contains just one
log. If there are 105 logs in the pile, how many layers are there? A
a. 12
c. 11
b. 14
d. 10
Find the sum of all integers between 84 and 719 which are exactly divisible by 5. A
a. 50 800
c. 23 780
b. 30 400
d. 45 680
A swimming pool is constructed in the shape of two partially overlapping circles, each of radius
9 m. If the center of each circle lies on the circumference of the other, find the perimeter of the
swimming pool. AG
a. 56.5 m
c. 85.7 m
b. 96.8 m
d. 75.4 m
A receiver in a parabolic television dish antenna is 3 feet from the vertex and is located at the focus.
Assuming that the dish is directed upward and the vertex is at the origin, find the equation of the
cross section of the reflector. AG
c. y2 = 12x
a. x2 = 12y
b. x2 = 8y
d. y2 = 8x
How many members between 3 000 and 5 000 can be formed from the digits 0, 1, 2, 3, 4, 5 and 6
of repetition is not allowed? SP
a. 160
c. 512
b. 420
d. 240
A sample of 5 is to be chosen from a batch of 6 resistors and 9 transistors. If the selection is made
randomly, what is the probability that the sample consists of 3 resistors and 2 transistors? SP
a. 240/1 001
c. 270/1 200
b. 480/1 000
d. 510/1 100
An organization has 25 members, 4 of whom are ECE. In how many ways can a committee of 3 be
formed so as to include at one ECE? SP
a. 970
c. 850
b. 350
d. 325

Mathematics

8
9
10
11
12
13
14
15
16
17

A snowball is being made so that its volume is increasing at the rate of 8 ft3/min. Find the rate of
change at which the radius is increasing when the snowball is 4 ft in diameter. DC
a. 0.159 ft/min
c. 0.259 ft/min
b. 0.325 ft/min
d. 0.015 ft/min
In a deck of 52 playing cards, what is the probability of drawing a number card or a club? SP
a. 9/13
c. 21/52
b. 1/26
d. 43/52
In a certain party, each of the group drinks coke or beer or whisky or all. Also 400 drink coke, 500
drink beer and 300 drink whisky, 100 drink coke and beer and 200 drink beer and whisky. One
who drinks whisky does not drink coke. How many are in the group? A
a. 900
c. 600
b. 300
d. 500
A man is riding his car at rate of 30 km/hr towards the foot of a pole 10 m high. At what rate is he
approaching the stop at 40 m from the foot of the pole? DC
a. 6.78 m/s
c. 8.08 m/s
b. 5.60 m/s
d. 4.08 m/s
A hut has a parabolic cross-section whose height is 30 m and whose base is 60 m. If a ceiling 40 m
wide is to be placed inside the hut, how high will it be above the base? AG
a. 19.85 m
c. 14.47 m
b. 16.67 m
d. 15.48 m
Find the area of a regular five-pointed star inscribed in a circle of radius 20 cm. AG
a. 559 cm2
c. 449 cm2
b. 349 cm2
d. 459 cm2
From a point on a level ground, the angles of elevation of the top and bottom of a PLDT tower,
situated on the hill and measured as 48 and 40, respectively. Find the height of the hill if the height
of the tower is 116 feet. T
a. 347.56 ft
c. 258.96 ft
b. 368.36 ft
d. 358.49 ft
Find the area of a region bounded by y = x2 5x + 6, the x-axis and the vertical lines x = 0 and
x = 4. IC
a. 16/3
c. 17/3
b. 5/7
d. 9/2
If n(A) = 115, n(B) = 326, n(A B) = 47 then n(A B) is equal to _____. A
a. 278
c. 373
b. 168
d. 162
An urn contains 5 white and 7 black balls. Another urn contains 3 white and 9 black balls. If one ball
from the first urn is selected at random and is transferred to the second urn, what is the probability
that the ball drawn from the second urn gives a black? SP

Mathematics
a. 125/326
b. 115/126

18
19
20

3
c. 151/216
d. 115/156

A fruit vendor goes to the market to buy fruits for resale at her store. She spend half her money for
mangoes, and one-third of what she had remains for bananas. She spent 150 for other fruits and
still has 200 left from the amount she originally had. How much money she have at the start? A
a. 1 050
c. 1 250
b. 1 500
d. 5 100
A sphere of radius 5 cm and a right circular cone of base radius 5 cm and height 10 cm, stands
on the table. How far from the table should the two solids be cut in order to have equal circular
sections? T
a. 2 cm
c. 5 cm
b. 4 cm
d. 7 cm
If (x + 3, 4 y) = (1, 7), then (x 3, 4 + y) is equal to _____. A
a. (4, 1)
c. (5, 1)
b. (5, 1)
d. (6, 1)

A
1

Answers

(b) Hint: Sum of an Arithmetic Progression


n
Sn = [2a1 + (n 1)d]
Eq.A1
2
where a1 = first term; d = common difference; n = number of terms; Sn = sum of n terms. Eq.A1 is
used for the sum of a certain number of an arithmetic sequence.

Given: Sn = 105 (total logs in pile); a1 = 1 (first pile); d = 1 (each layer of the pile is incremented
by 1)
Required: n (number of layers)
n
105 = [2(1) + (n 1)(1)]
2
210 = 2n + n2 n
0 = n2 + n 210
Apply the quadratic formula and choose a positive integer.
n = 14

(a) Hint: n Term and Sum of an Arithmetic Progression

n
Sn = (a1 + an)
Eqs.A2 & A3
2
where an is an n term (and can be referred as a last term). Eq.A2 is used if there is a part of n term
without summing that sequence. Eq.A3 is definitely the same with Eq.A1 by simply substituting an
from Eq.A2.
an = a1 + (n 1)d

and

Given: a1 = 85; d = 5; an = 715


Required: Sn
Find first the value of n.

715 = 85 + (n 1)(5)
n = 127
Finally,

127
Sn = (85 + 715) = 50 800
2

(d) Hint: Sector and Circumference of a Circle


s = r

and

C = 2r

Eqs.A4 & A5

where s = arc length, r = radius, = angle of the sector, C = circumference. These formulas are
commonly applied in a perimeter of a circle.
Given: Fig.A1a shows the problem.
Required: perimeter P of the non-overlapping circle
The angle and the arc length s are unknown. Since the overlap touches the center of two circles, an
equilateral triangle can be drawn and is always 60 of each angle (Fig.A1b). To complete the angle
of the arc length, it is 120.
Note that for the arc length, the angle which is in degree is converted to a radian.

Mathematics

5
s = 9(120)(/180) = 6

60

r=9m

(a)
Fig.A1

(b)

To get the perimeter, the one that does not overlap, subtract the arc length from the circumference.
P = C s = 2(9) 6 = 12
Doing the other circle is same, so add the total perimeter.
PT = 12 + 12 = 24 m or 75.4 m

(a) Hint: Formula of an Upward Parabola


x2 = 4ay

Eq.A6

where a is a focus. Eq.A6 describes the parabola faces upward and its vertex is located at the origin.
Given: a = 3
Required: parabolic equation
x2 = 12y

(d) Hint: Permutation and Simple Probability (Another Solution)


n!
nPr =
(n r)!
where the symbol ! is a factorial. Eq.A7 is used when the situation is arranged in order.

Eq.A7

Given: numbers 3 000 and 5 000


Required: possible numbers between 3 000 and 5 000 having the digits 0 1 2 3 4 5 6 without repetition
Two solutions are as follows:
1) These numbers consist of 4 digits. The first place starting at the left may be filled in 2 ways, that
is by digits 3 and 4. The remaining 6 digits, which are 0 1 3 4 5 6, may be arranged in the 3 other
places writing 6P3. The solution will be formed as
2(6P3) = 240
2) By not using Eq.A7, have four blanks to fill the numbers as to follow the rules. This solution is
called a simple probability. Again, the first place starting at the left is filled in 2 ways: 3 and 4.
2___
The second place is filled with 6 which is considered as the highest number given.
26__
The third place is filled with 5 because the rule is not repeating.
265_

Mathematics
The last place is 4 as the same reason from the previous step.
2654
Multiply all digits.

2(6)(5)(4) = 240

(a) Hint: Combination and Probability

n!
S
Eqs.A8 & A9
nCr = and PE =
r!(n r)!
T
where PE = probability that the event will happen, S = successful outcomes, T = total number of
possible outcomes. Eq.A8 is used when the situation does not matter the order; in other words, it is
a group of random. Eq.A9 is the numerical measure of the likelihood of an event to occur.
Given: 6 resistors and 9 transistors
Required: chance to pick 3 resistors and 2 transistors
The two kinds of object with the number of picks are expressed in combination and multiplied.
6C3 9C2
Importantly the total objects and picks are also expressed in combination.
15C5
With S = 6C3 9C2, and T = 15C5, then:
6C3 9C2
= 720/3 003 = 240/1 001
15C5

(a) Hint: Combination (See Eq.A8.)


Given: 25 members having 4 ECEs
Required: ways of 3 committees to include 1, 2 or 3 ECEs
Two solutions are as follows:
1) There are 21 non-ECEs. Listing the chance of ECE who will be a committee is expressed in
combination. The first step solution might be 1 ECE who could be a committee and the remaining
non-ECEs would be 2 committees, and so on:
At least 1 ECE: 4C1 21C2 = 840
At least 2 ECEs: 4C2 21C1 = 126
At least 3 ECEs: 4C3 21C0 = 4
Add the total ways.
2) Do the shortest method.

840 + 126 + 4 = 970


25C3 21C3 = 970

(a) Hint: Volume of a Sphere and Derivative

4
Eq.A10
V = r3
3
The derivative is applied when the function deals about the rate of change. Most common cases of
the rate of change are affected by time t.
Given: d (diameter) = 4 ft; r = 2 ft; dV/dt = 8 ft3/min (rate of change of the volume)
Radius is half of a diameter.

Mathematics

Required: dr/dt (rate of change of the radius)


Take the derivative on both sides.

4
d
V = r3
3
dt
dV 4
dr
dr
= (3)r2 = 4r2
dt 3
dt
dt
dr
8 = 4(2)2
dt
dr
= 0.159 ft/min
dt

(d) Hint: Mutually Inclusive Events of Probability


P(A or B) = PA + PB P(A and B)

Eq.A11

Two or more events are said to be inclusive, when one or the other or both can occur (Eq.A11).
Given: a deck of cards
Required: chance to draw a number card (where ace is included), or a club (shape )
A deck consists of 13, 13, 13, and 13 with the total of 52 cards. Each shape has A, 2, 3, 4, 5,
6, 7, 8, 9, 10, J, Q and K. Through distinction, there are 40 number cards (in any shape) and 13 cards
of . It is impossible that there will be 53 cards in all. To reduce the excess, it makes sense that
40 number cards have 10 cards of . Express them in a final form where PA = 40/52, PB = 13/52,
and P(A and B) = 10/52.
40 13 10 43
+=
52 52 52 52

10

(a) Hint: Sets


Given: 400 drink coke; 500 drink beer and 300 drink whisky; 100 drink coke and beer and 200
drink beer and whisky; one who drinks whisky does not drink coke
Required: total groups
The common illustration for sets is a Venn diagram. To represent it, the groups are sorted according
to their drinks (Fig.A2). All drinks are counted no more, no less.
beer
coke
300
400 100

100

200

200 whisky
100

500 200 100

300 200

Fig.A2
Add the groups.

300 + 100 + 200 + 200 + 100 = 900

11

Mathematics
(c) Hint: Pythagorean Theorem and Derivative
c2 = a2 + b2

Eq.A12

where c = hypotenuse, and a and b are the two sides (adjacent and opposite) of the triangle. Eq.A12
is used only for a right triangle.
Given: 10-meter pole; x = 40 m (road); dx/dt = 30 km/hr or 8.33 m/s (initial rate)
For the consistence of units, convert km/hr to m/s.
Required: rate of velocity at the final road
rate of 8.33 m/s

10 m pole

10

40

x = 40 m
Fig.A3
Solve the hypotenuse c.

c2 = 402 + 102

c = 402 + 102 = 41.23

To apply the derivative in Eq.A10, let x varies where the car is displaced.
d
(c2 = x2 + 102)
dt
The derivative of constant is zero.
dx
dc
2c = 2x + 0
dt
dt
dc x dx
40
= = (8.33) = 8.08 m/s
dt c dt 41.23

12

(b) Hint: Parabola and Proportion


Proportion is one of the application to measure between two similar shapes with different sizes.
y1/x1 = y2/x2

Eq.A13

where the subscripts represent the shape 1 and shape 2.


Given: Fig.A4a shows the problem. Assume that the downward parabola faces to the positive axis.
Required: height h between the base and the ceiling
origin
30 m
h

30 h
ceiling

40 m
y

base

60 m
(a)
Fig.A4

30

20
30
(b)

Mathematics

In Eq.A6, x is squared. The two similar shapes (Fig.A4b) are selected to solve the height h.
30 30 h
=
302
202
400(30) = 900(30 h)
h = 16.67 m

13

(c) Hint: Sine Law and Area of a Triangle, and Area of a Five-Pointed Star (Another Solution)
Eqs.A14 & A15
a
b
c
sin A sin B sin C
= = and = =
sin A sin B sin C
a
b
c
where a, b and c are the sides of a triangle, and A, B and C are the angles. These formulas are used
if the triangle is a not right triangle.
Eq.A16
1
A = (shortest side)(base) sin
2
Eq.A16 is an area of a triangle in terms of an angle. If there is a height, the shortest side and sin
are substituted into it.
Eq.A17
A = 1.123 r2
where r = radius. The most efficient solution is the area of a five-pointed star itself (Eq.A17).
Given: Fig.A5a shows the problem.
Required: area of the star
Two solutions are as follows:
1) The star is sliced into 10 triangles symmetrically; however the circle is insignificant. One triangle
in enlarged scale is selected, and labeled as a model-based (Fig.A5b). It does not matter where
the sides of a triangle are labeled as long as the angles are followed accordingly.
(a) (b)

r=2

0 cm

C
b

a
B

c=r

Fig.A5
A revolution around 10 triangles (or of a circle) is 360. Divide it by 10 triangles to get the angle B.
B = 360/10 = 36
Notice that the angle A is half of the angle B due to symmetry; so A = 18. To find the angle C,
the total angle of a triangle is always 180 and subtract it from the known angles.
C = 180 36 18 = 126
Use sine laws to solve the unknown sides:
a
20
=
sin 18 sin 126

10

Mathematics
a = 7.64
and

b
20
=
sin 36 sin 126
b = 14.53

The area of a star is 10 times the area of that triangle.


A = 10A = 10(1/2)(ac sin B)
= 10(1/2)(7.64)(20) sin 36 = 449 cm2
From the solution above, the angle B is the angle that meets the shortest side a and base c.
2) Using Eq.A17,

14

A = 1.123(20)2 449 cm2

(d) Hint: Trigonometric Function


Functions such as sine, cosine and tangent are common use to measure the length of a right triangle.
Remember the ratios of two sides of a right triangle which defines from each trigonometric function.
Given: Fig.A6a shows the problem.
Required: height of the hill
116 ft
116 + h
h

h
point

40 48

40

x
(a)
Fig.A6

48
x

x
(b)

Finding the hypotenuse is not necessary, The tangent is a ratio of the opposite and adjacent side,
and is expressed of two triangles on Fig.A6b:
116 + h
h
tan 48 = and tan 40 =
x
x
Equate two solutions above as one, since both x are the same.
116 + h
h
x==
tan 48 tan 40
(tan 40)(116 + h) = (tan 48)h
h = 358.49 ft

15

(c) Hint: Definite Integral


A definite integral is one of the application to solve area of curves or any asymmetric geometry.
b
a

f(x) dx

Eq.A18

Mathematics

11

where f(x) is a function, and a and b are the boundary values (or limits); the boundary value b must
be greater than a.
Given: function y = x2 5x + 6 with vertical lines at x1 = 0 and x2 = 4
Required: area of all regions
Predicting and drawing a graph of a function (Table A1 and Fig.A7) are needed for easy to find out
the boundary values and the areas.
y
y = x2 5x + 6
x
y

0
6

1
2

2
0

3
0

4
2

A2
A1
0

Table A1

A3
3 4

Fig.A7

Three shaded parts are the areas bounded by the curve, two vertical lines and x-axis. Identify the
boundary values of each area. Integrating A1 yields:
2

y dx =

(x2 5x + 6) dx

2
x3 5x2
= + 6x
2
3
0
3
03 5(0)2
2 5(2)2
= + 6(2) + 6(0)
2
2
3
3
= 14/3

From the solution above, the steps are as follows: (1) Integrate the function and set aside the limits at
the right; (2) substitute the upper limit and lower limit; (3) and subtract the second result from the first
result.
Integrating also A2 and A3 at different limits yields:
3

and

y dx =
4
3

y dx =

(x2 5x + 6) dx = |1/6| = 1/6


4
3

(x2 5x + 6) dx = 5/6

From the solved A2, area does not occur negative integer.
Add the total area.

AT = 14/3 + 1/6 + 5/6 = 17/3

To solve the area as an alternative way, this short method below is wrong because the area A2 is
located on the negative y-axis (fourth quadrant). If ever all areas are located on the positive quadrant
depending of the function, that alternative way may be used.
4

y dx =

(x2 5x + 6) dx = 16/3

12

16

Mathematics
(c) Hint: Sets
A set is a collection of objects or numbers. When two or more sets are met, this is expressed in a
notation or drawn with a Venn diagram. The symbols used are such as union and intersection .
Given: n(A) = 115; n(B) = 326; n(A B) = 47
Required: n(A B)
Understanding the law of sets, set n(A B) defines literally that the part of set n(A) remains by taking
away the intersection of sets n(A) and n(B). Express into an algebraic form:
n(A B) = n(A) n(A B)
The union of sets is similar to addition. Instead set n(A B) becomes the total sets of n(A) and n(B)
but their intersection is taken away due to set n(A B). Express also into an algebraic form:
n(A B) = n(A) + n(B) n(A B)
Using the two equations above yields:

47 = 115 n(A  B)
n(A  B) = 68

and

17

n(A B) = 115 + 326 68 = 373

(d) Hint: Dependent Events of Probability


P(E1E2) = [PE1][P(E1/E2)]

Eq.A19

where P(E1E2) = the probability that both event E1 and E2 occur, P(E1/E2) = the conditional probability
of E2 given that E2 has occured. Two events are dependent if the occurrence or non-occurrence of
one affect probability of the occurrence of the other (Eq.A19).
Given: the first urn contains 5 white balls and 7 black balls; the second urn contains 3 white balls
and 9 black balls
Required: chance to pick one black ball in the first urn; then transfer it to the second urn
If you think it is a white, the probability is:
Pw = (5/12)(9/13)
The probability form 9/13 means that there are still 9 black balls in the second urn when transferring
a white ball.
If you think it is a black, the probability is:
Pb = (7/12)(10/13)
So there would be 10 black balls in the second urn.
Add the total chances.

18

PT = Pw + Pb = 115/156

(a) Hint: Algebraic Expressions


When dealing with word problems, the algebraic phrases are translated into math expressions. The
unknown number is usually represented in a variable such as x.
Given: Reread the problem.
Required: the amount of the original money
As to analyze the problem, the following phrases are translated:

Mathematics

13
x = be the original money
x
= cost of mangoes
2

1
x
x = cost of bananas
3
2
150 = cost of other fruits
200 = money left

All fruits that she spent are subtracted from the original money until she got 200.
x 1
x
x x 150 = 200
2
2 3
x = 1 050

19

(a) Hint: Pythagorean Theorem and Proportion


Given: Fig.A8 shows the problem.
Required: height of the equal cut section of a sphere and cone
equal height

10 cm

5 cm
h

surface level

5 cm
Fig.A9

Create a triangle in the sphere (Fig.A9a) and get the radius x of the cut using Pythagorean Theorem.
In the cone, create two triangles (Fig.A9b) and get the radius x of the cut using proportion.

5h

10

10 h

x
5

and

(a)
Fig.A9
a2 = c2 b2
x2 = 52 (5 h)2

x = 10h h2

(b)

14

Mathematics

Equate them as one.

10 10 h
=
5
x
10 h
x=
2
2 10 h
10h h =
2
2
10 h
10h h2 =
2
(10 h)(10 h)
(10 h)h =
4
h = 2 cm
x=

20

(b) Hint: Basic Algebra Technique


Given: (x + 3, 4 y) = (1, 7)
Required: (x 3, 4 + y) = ?
Transpose the variables where x + 3 = 1, and 4 y = 7.
x = 2 and y = 3
Substitute the known x and y on the requirement.
2 3 = 5 and 4 + (3) = 1

15

Quick Tips

Welcome to this section! In normal mode, solving an engineering problem is spent less than or within two
minutes. Very proficient you are if you solve a hard and challenging problem on the said time. Because time
is counting on you and becomes your worst enemy, be alert especially when you take a licensure exam. Of
course, your best buddy is the calculator.
Modern scientific calculators have so many functions and operations that help to eliminate of some memorized
formulas and/or to shorten the solution. One of the common brand scientific calculators is Casio with a
model: fx-991ES PLUS (or fx-991ES which is the previous one). So we are going to use it for the rest of this
document. Be familiar with it as usual.

The Shift-Solve Technique


Here, the equation must contain one representation of variables in a polynomial term. The aim is to find the
unknown variable. Lets choose an equation in Item 19, that is
x 1
x
x x 150 = 200
2 3
2
You see that the variable represents as x. Rather than you solve manually such as transposing and grouping
the x variables into one variable, use the calculator. The shift-solve technique is used when inputting an
equation. We know that it is composed of left- and right-hand between the equal sign (=).
Line Format Equation
1) If you want to change that equation into a line format, this way would be like
x (x 2) (1 3)(x (x 2)) 150 = 200
For variable x, the keys are Q); do not press them at the same time. The instruction might be redundant
if you are guided word by word. Based on our equation above, simply follow these consecutive keys:
Q)-(Q)P2)-(1P3)(Q)
-(Q)P2))-150Qr200
Caution: Do not use the key p for the equal sign of the equation because that key is used to calculate for
the output (or answer). Instead press Qr.
As you input them, the screen below is provided.

2) To solve x, press qr which is the shift-solve. A command appears as Solve for X. Then finally
press p.

16

Mathematics

Math Format Equation


1) If you want a precise and neat format, follow these consecutive keys:
Q)-aQ)R2$-a1R3$(Q)
-aQ)R2$)-150Qr200
The screen below is provided.

Note: Math format equation is used in COMP (Compute) mode; the keys are w1. However, CMPLX
(Complex) mode is not applicable with this technique.
2) Do the shift-solve.

Disadvantages of Shift-Solve
For some reasons, getting the output takes a few seconds to wait, or the calculator cant solve.
1) The variable is placed on the denominator using math format equation.
2) The variable has a power (or degree) in a polynomial term.
3) Handles more mathematical functions (such as trigonometric, logarithmic, exponential form, etc.)
4) Other than a linear equation such as quadratic and quartic equation, one root will exist. The calculator
cant solve if the root would be a complex number (not real number).
5) The calculator cant solve inseparable variables (commonly non-linear equation), and expressions with
complex numbers.

Quadratic Equation
Here, do not apply the quadratic formula to extract the roots of the quadratic equation. The calculator is readyto-go for you to save time and effort.
The general form of the quadratic equation is
ax2 + bx + c = 0
where a, b and c are coefficients. Lets choose an equation in Item 1, that is

Mathematics

17
n2 + n 210 = 0

1) Go to w5 (EQN, Equation). Then press 3 for the general form of the quadratic equation. You see
that the zeroes place on each coefficient respectively.

2) Since n is a variable, let x2 + x 210 = 0, where a = 1, b = 1, c = 210. To input these coefficients, press
1p1p-210
and you have this process as shown below.

3) Press p to get the roots. The root x1 is first shown up; for root x2, press R. The arrowhead on the upper
screen indicates that there is a previous and/or next output. (Its purpose is known also as history.)

There is also a quartic equation in the calculator. Find it by pressing w54.

Definite Integral
You dont need to integrate manually such as integration by parts, trigonometric substitution, integration
by substitution, etc. The calculator can solve definite integral. However, do not expect that you are going to
input integrals without limits here. We are going to create a math format equation for it. Lets choose the first
definite integral in Item 15, that is
2
0

(x2 5x + 6) dx

1) Press y for integral function. Then follow these consecutive keys:


E2R0!!Q)f2$-5Q)+6

18

Mathematics

2) Press p to get the answer.

Storing of Data
This technique helps you to collect the outputs and then to use them for the another math expression. This is
also important if the outputs value is a large number, (terminating or non-terminating) decimal number, and
fractional number.
1) Lets choose the output in the Definite Integral Section. We store it by replacing a variable. Press qJ
for STO (storage), and then z for replacing it with variable A.

2) Do it for the remaining definite integrals:


3
2

(x 5x + 6) dx
2

and

4
3

(x2 5x + 6) dx

and store them as variables B and C.

3) Based on the solution, add the variables by pressing


Q)-Qx+Qcp
We put minus sign for B since the area must not be a negative value. You have now the answer.

Arithmetic Progression
Aside from using Eq.A2, you calculate either the number of terms n, or the nth term an in a statistical way.
Lets choose the given in Item 2 as which they will form an arithmetic progression. Recalling a1 = 85, d = 5,
and an = 715.
1) Press w3 (STAT, statistics). Then press 2 for linear function. We use that because the arithmetic
progression becomes a linear function.

Mathematics

19

You will see a table function on the screen.

2) Since 85 is the first term, then n = 1. Because d = 5, 85 is next to 90, and 90 is the second term, so n = 2.
Input them as shown below.

3) Press C to exit. The data you inputted is saved.


Caution: Do not press W because everything that you have inputted will be lost. If happens, then you
input again.
Press q1 for STAT controls, and then press 5 (Reg, regression). Because the unknown value is
the number of terms n with the given an = 715, press 4 for x , (x-hat or x-regression).

That symbol appears alone at the upper-left, and then type 715 before it. Then press p.

20

1
2
3
4
5
6
7
8
9

20

Mechanics, Thermodynamics,
and Engineering Economics

Mechanics (Pretest: 15, Post-test: 613)

A railroad flatcar of mass 2 000 kg, rolls to the right at 10 m/s and collides with a flatcar of mass
3 000 kg which is rolling to the left at 5 m/s. The flatcars couple together. Their speed after the
collision is _____.
a. 1 m/s
c. 5 m/s
b. 2.5 m/s
d. 7 m/s
A boy is standing on an elevator which is traveling downward with a constant velocity of 30 m/s.
He throws a ball vertically upward with a velocity of 10 m/s, relative to the elevator. What is the
velocity of the ball, magnitude and direction, relative to the elevator shaft the instant the boy releases
the ball?
a. 20 m/s, down
c. 40 m/s, down
b. 20 m/s, up
d. 40 m/s, up
One newton is equivalent to _____.
a. kg-m/s
b. kg-m/m/s

c. kg-m/s/s
d. m/s2/kg

Momentum is the mass and _____.


a. acceleration
b. velocity

c. force
d. time

A machine perform 8 joules of work in 2 seconds. How much power is delivered by this machine?
a. 4 W
c. 8 W
b. 6 W
d. 12 W
A train is traveling 20 km/hr. It is 0.5 km behind you. You want to beat the train to a crossing that
is 0.5 km away. How fast must you run?
a. 4 km/hr
c. 15 km/hr
b. 5 km/hr
d. 10 km/hr
From the speed of 100 km/hr, a car decelerates at the rate of 15 m/min/sec along a straight road.
Which of the following gives the distance traveled at the end of 40 sec?
a. 94.111 m
c. 455.56 m
b. 911.11 m
d. 3 800 m
A person pushes a box across a horizontal surface at a constant speed of 0.5 m/s. The box has a
mass of 40 kg and the coefficient of sliding friction is 0.25. The power supplied to the box by the
person is _____.
a. 0.2 W
c. 100 W
b. 50 W
d. 5 W
A fighter plane flies over the ocean in level flight at an altitude of 19.6 meters. How long does it take
a bomb dropped from its wing to impact the water?

Mechanics, Thermodynamics, and Engineering Economics


a. 1 sec
b. 0.5 sec

10
11
12
13
14
15
16
17
18

21

c. 2 sec
d. 2.83 sec

An artillery is launched upward at an angle of 44 with a muzzle velocity of 40 m/s. What is the
time that the artillery would reach the highest point of its flight?
a. 6.28 sec
c. 5.66 sec
b. 3.82 sec
d. 2.83 sec
Of the following terms, which is the product of force and time?
a. energy
c. work
b. impulse
d. power
Three forces that act on a particle are given by F1 = 20i 36j + 73k N, F2 = 17i + 21j 46k N,
and F3 = 12k N. Find the magnitude of the resultant force.
a. 16.7 N
c. 30.2 N
b. 21.4 N
d. 15.5 N
A 5-meter uniform plank of mass 100 kilograms rests on the top of a building with 2 meters extended
over the edge. How far can a 50-kg person venture past the edge of the building on the plank before
the plank just begins to tip?
a. 0.67 m
c. 0.5 m
b. 1 m
d. 2 m
Thermodynamics (Pretest: 1418, Post-test: 1923)

_____ is the most efficient thermodynamics cycle.


a. Diesel
c. Carnot
b. Brayton
d. Otto
The maximum efficiency of a heat engine that operates between temperature of 1 500 K in the
firing chamber and 600 K in the exhaust chamber is mostly nearly _____.
a. 33%
c. 60%
b. 40%
d. 70%
How many molecules are present in 2.5 L of gas at STP?
a. 5.6 1022
c. 6.0 1022
b. 1.5 1022
d. 6.7 1022
An ideal gas confined in a box initially has pressure p. If the absolute temperature of the gas is
doubled and the volume of the box is quadrupled, the pressure p is _____.
a. p/8
c. p/2
b. p/4
d. p
On a day when the barometer reads 755 mmHg, a tire pressure gauge reads 205 kPa. The absolute
pressure of the tire is _____.
a. 100 kPa
c. 1.54 mmHg
b. 204 kPa
d. 2.29 mHg

22

19
20
21
22
23
24
25
26
27
28

Mechanics, Thermodynamics, and Engineering Economics


The volume of a gas under constant pressure increases or decreases with temperature is called _____.
a. Charless Law
c. Gay-Lussacs Law
b. Boyles Law
d. Ideal Gas Law
At one atmosphere, a cylinder with movable piston contains 2 000 mL of gas. If the pressure is
increased to 5 atmospheres and the temperature remains unchanged, what is the new volume of the
gas?
a. 10 000 mL
c. 4 000 mL
b. 400 mL
d. 1 000 mL
At what point is the temperature the same on the Celsius and Fahrenheit scales?
a. absolute zero
c. zero
b. negative 40
d. never
Which of the laws of thermodynamics is often invoked to discredit attempts at perpetual motion?
a. Zeroth Law of Thermodynamics
c. First Law of Thermodynamics
b. Third Law of Thermodynamics
d. Second Law of Thermodynamics
At STP, the volume of the gas is 1 000 mL. If the temperature is increased to 25C and the pressure
is doubled, what is the new volume of the gas?
a. 500 mL
c. 546 mL
b. 273 mL
d. 1 000 mL
Engineering Economics (Pretest: 2433, Post-test: 3436)

_____ is the payment for the used of borrowed money.


a. Loan
c. Maturity value
b. Interest
d. Principal
_____ is the type of annuity where payments continue infinitely.
a. Perpetuity
c. Annuity due
b. Deferred annuity
d. Ordinary annuity
2 000 is borrowed for 105 days at 12% per annum simple interest. How much will be due at the
end of 105 days?
a. 2 070
c. 2 270
b. 2 170
d. 2 370
The amount of 20 000 was deposited in a bank and was earning an interest of 6.5% per annum.
Determine the total amount at the end of 17 years if the principal and interest were not withdrawn
during this period.
a. 53 480.93
c. 58 403.93
b. 54 830.93
d. 58 340.93
What nominal rate, compounded semiannually, yields the same amount as 25% compounded
quarterly?
a. 25.78%
c. 28.57%
b. 27.58%
d. 24.78%

Mechanics, Thermodynamics, and Engineering Economics

29
30
31
32
33
34
35
36

23

Find the present worth of a future payment of 290 000 to be made in 4 years with an interest of
9% compounded annually?
a. 254 043.31
c. 205 344.31
b. 245 403.31
d. 205 443.31
A sum of 15 000 is invested now and left for eight years, at which time the principal is withdrawn.
The interest has accrued and is left for another eight years. If the effective annual interest rate is 15%,
what will be the withdrawal amount at the end of the 16th year?
a. 94 487.97
c. 94 748.97
b. 94 478.97
d. 98 447.97
How many years will take money to quadruple if it earns 10% compounded annually?
a. 12
c. 14
b. 13
d. 15
What is the present worth of a 3 year annuity paying 3 000 at the end of each year, with interest
at 8% compounded annually?
a. 7 654.04
c. 7 420.89
b. 7 731.29
d. 7 590.12
An amount of 2 340 is deposited in a bank paying an annual interest rate of 3.1%, compounded
continuously. Find the balance after 3 years.
a. 2 568.06
c. 2 586.06
b. 2 658.06
d. 2 685.06
_____ is the series of equal payment made at the equal intervals of time.
a. Loss
c. Liability
b. Sunk cost
d. Annuity
What annuity is required over 12 years to equate with a future amount of 20 000? Assume i = 6%
annually.
a. 1 205.74
c. 1 290.34
b. 1 185.54
d. 1 107.34
A loan of 170 000 is to be paid in three years at the amount of 185 000 compounded annually.
What is the effective rate of money?
a. 5.859%
c. 3.859%
b. 2.859%
d. 4.859%

24

24

Answers

(a) Hint: Completely Inelastic Collision


m1v1 + m2v2 = (m1 + m2)vf

Eq.B1

where m1 = mass of the first object, m2 = mass of the second object, v1 = velocity of the first object,
v2 = velocity of the second object, vf = final velocity after the collision. Eq.B1 is a collision in which
the total kinetic energy after the collision is less than before the collision.
Given: m1 = 2 000 kg; v1 = 10 m/s; m2 = 3 000 kg; v2 = 5 m/s (opposite direction)
Required: vf
2 000(10) + 3 000(5) = (2 000 + 3 000)vf
vf = 1 m/s

(a) Hint: Relative Velocity in One Dimension


This item involves equilibrium when particle is at rest or moving with constant velocity in an inertial
frame of reference.
Given: velevator = 30 m/s (constant); vball = 10 m/s
Required: velocity of the ball relative to the elevator
Simply sum up the two velocities since they are moving vertically.
30 + 10 = 20 m/s or 20 m/s, down
The ball is actually not moving up if you observe the elevator and ball of that motion.

3
4
5

(b) Hint: Basic Algebra Technique


One newton is equivalent to kilogram times meter per second squared (kg m/s2). The answer is
kg-m/s/s (Letter c) where the fractional form of m/s and s is being reciprocal and then multiplied.
(b) Momentum is the product of mass and velocity.
(a) Hint: Power (in Relationship of Work and Time)
W
P=
t
Power is a time rate at which work is done.

Eq.B2

Given: W = 8 J; t = 2 s
Required: P
P = 8/2 = 4 W

(d) Hint: Velocity and Proportion

d
v=
t
where d is distance. Velocity is a time rate at which the particle is in motion.
Given: Fig.B1 shows the problem.
Required: your speed to beat the train to the crossing

Eq.B3

Mechanics, Thermodynamics, and Engineering Economics


crossing

you

25

v = 20 km/hr
train

0.5 km

0.5 km
Fig.B1

The two velocities are expressed as:

0.5 + 0.5
vtrain = 20 =
t

and

0.5
vyou =
t

You and the train travel at the same time, so equate two solutions above.
0.5 + 0.5 0.5
t==
20
vyou
vyou = 10 km/hr

(b) Hint: Constant Acceleration


Acceleration describes how the velocity changes with time. If an acceleration is constant, in this case,
the velocity changes at the same rate. If the particle accelerates constantly along x-axis, four equations
relate as follows:
vx = v0x + axt
Eq.B4a
1 2
x = x0 + v0xt + axt
Eq.B4b
2
vx2 = v0x2 + 2ax(x x0)
Eq.B4c
v0x + vx
x x0 = t
Eq.B4d
2
where x0 = initial position, x = final position, v0x = initial x-velocity, vx = final x-velocity, ax = xacceleration, t = time.
Given: v0x = 100 km/hr or 27.78 m/s; x0 = 0 m; t = 40 s; ax = 15 m/min/s or 0.25 m/s2
For consistence, some units of the quantities are converted. Most initial velocities start at zero. Note
that the deceleration is a negative value. Proofing that unit of acceleration shows:
m 1
1 min
= 0.25 m/s2
15
min s
60 s
Required: x
Select Eq.B4b which satisfies the following given.
x = 0 + 27.78(40) + (1/2)(0.25)(402) = 911.11 m

(b) Hint: Power and Relationship of Other Quantities such as Work and Frictional Force
W = Fs

Eq.B5

where W = work, F = force, s = displacement. The work done by a constant force on a particle that
moves a displacement is a product (Eq.B5).
F = 0

Eq.B6a

26

Mechanics, Thermodynamics, and Engineering Economics


where F is a force vector. When particle is at rest or moving with constant velocity in an inertial frame
of reference, the net force (that is, the vector sum of all forces) acting on it must be zero (Eq.B6a).
Fx = 0

and

Fy = 0

Eqs.B6b & B6c

The equation in component form is often used (Eq.B6b and Eq.B6c).


Common types of force are normal force, friction force, and weight.
Eq.B7
Eq.B8

w = mg
f = n

where w = weight, m = mass, g = acceleration of gravity, f = friction force, = coefficient of friction,


n = normal force. Weight is the gravitational force that the earth exerts on the particle; it is directly
proportional to its mass (Eq.B7). Friction force is a force exerted on a particle by a surface that acts
parallel to the surface, in the direction that opposes sliding; it is about proportional to the normal
force (due to the type of surfaces material) (Eq.B8).
Given: Fig.B2a shows the situation with the following given quantities: m = 40 kg; g = 9.8 m/s2
(constant) or 10 m/s2 (round-off); v = 0.5 m/s (constant); = 0.25 (unitless)
Required: power applied by the boy
y
v
n

Fa

(a)
Fig.B2

(b)

Fig.B2b illustrates free-body diagram for box. Deriving the power yields:
W Fad
P==
= Fav = nv = mgv
t
t

From Eq.B5, distance


can be referred from
displacement.

From Eq.B3

By using Eq.B6c,
By using Eq.B6b,
applied force Fa is equal normal force is equal to
weight.
to friction force.
Fy = n w = 0
Fx = Fa f = 0
n = w = mg
Fa = f = n

P = 0.25(40)(10)(0.5) = 50 W

(c) Hint: Freely Falling Particle


The acceleration of a falling particle moves constantly if air resistance is negligible. From Eq.B4b,
let the particle moves along y-axis and equation becomes:
1
y = y0 + v0yt gt2
Eq.B9a
2
1
y = gt2
Eq.B9b
2

Mechanics, Thermodynamics, and Engineering Economics

27

The y-acceleration is downward in negative y-direction, so ay = g. Eq.B9a is used if the particle acts
up-and-down motion while Eq.B9b is used if the initial velocity and initial position are zero.
Given: y = 19.6 m (height h is at negative y-direction); g = 9.8 m/s2 (constant)
Required: t
Select Eq.B9b.

19.6 = (1/2)(9.8)t2

t=2s

10

(d) Hint: Projectile Motion


If air resistance is negligible, the trajectory (or parabolic path) of a projectile is a combination of
horizontal motion with constant velocity and vertical motion with constant acceleration. Formulas
of constant x-acceleration and free fall are used in the concept of projectile motion. The component
of velocity is applied through trigonometry.
x = (v0 cos 0)t
Eq.B10a
1
y = (v0 cos 0)t gt2
Eq.B10b
2
vx = v0 cos 0
Eq.B10c
vy = v0 sin 0 gt
Eq.B10d
where 0 is an angle from the initial velocity.
Given: 0 = 44; v0 = 40 m/s; g = 9.8 m/s2 (constant)
Required: t at the maximum height
Reminding that the projectile (or even free fall) has zero vertical velocity when particle reaches at
the maximum height. Select Eq.B10d which the satisfies the following given including vy = 0.
0 = 40 sin 44 9.8t
t = 2.83 s

11
12

(b) Impulse is the product of force and time.


(b) Hint: Resultant Force and Its Magnitude
Eq.B11a
RF = F1 + F2 + F3 +
= (F1x + F1y + F1z) + (F2x + F2y + F2z) + (F3x + F3y + F3z) +
= (F1x + F2x + F3x + ) + (F1y + F2y + F3y + ) + (F1z + F2z + F3z + )
= RFx + RFy + RFz

Eq.B11b
RF = RFx2 + RFy2 + RFz2
where RF = resultant force,
F1, F2, F3, = force vectors,
F1x, F1y, F1z, and so on = component vectors of force vector,
(The magnitude, which is not shown in Eq.B11b, is denoted as F1x, F1y, F1z, and so on, and these
are called the components of force vector.)
RFx, RFy, RFz = component vectors of resultant force,
RF = magnitude of resultant force,
Rx, Ry, Rz = components of resultant force (also in magnitude).
Adding two or more vector quantities particularly such as force vectors is called a resultant vector. As

28

Mechanics, Thermodynamics, and Engineering Economics


to refer Eq.B11a, a vector consists of component vectors showing an expanded form; the terms are
arranged in x, y and z dimensions. To get its magnitude, Eq.B11b is used and derived from Pythagorean
Theorem.
Given: F1 = 20i 36j + 73k N; F2 = 17i + 21j 46k N; F3 = 12k N
where i, j, k are unit vectors similar with x-, y- and z-axis respectively.
Required: RF
RF = (20i 36j + 73k) + (17i + 21j 46k) + (12k)
= (20i 17i) + (36j + 21j) + (73k 46k 12k)
= 3i 15j + 15k N
The magnitude is:

13

RF =

32 + (15)2 + 152 = 21.4 N

(b) Hint: Moment of Force and Net Moment


M = Fd

and

M = 0

Eqs.B12a & B12b

Moment M is the tendency of force to cause or change a bodys rotational motion. It is a product of the
force F and the corresponding perpendicular distance d (Eq.B12a). If two or more forces are applied
in equilibrium, the net moment must be zero (Eq.B12b).
Given: Fig.B3 shows the problem having a 100 kg and 5-meter uniform plank, and 50 kg person.
Required: limited distance x of the person from the edge to maintain the plank balance
x

3m

2m
Fig.B3

edge of a building

There are no applied forces, so his and planks weight will be used. Imagine that the plank acts as a
seesaw, and the edge acts as a fulcrum. Because the axis of rotation is at the edge, the plank has two
weights even though the 3 m part of plank would not rotate downward on this situation. Get first the
two separated masses of plank by using ratios:
(3/5)100 = 60 kg and (2/5)100 = 40 kg
The weights (using Eq.B7) including the person are 588 N, 392 N and 490 N. Then the center of
gravity is introduced. The center of gravity of a linear body is located at the center. Solving x using
the net moment yields:
0 = 588(1.5) 392(1) 490x
0 = 60(1.5) 40(1) 50x
x=1m
From the solution above, let other forces negative based on the direction of rotation and the axis of
rotation. The gravity is canceled since there are no applied forces. Thus in this case, Eq.B12a can
be changed into M = md where m is mass.

14

(c) Carnot is the most efficient thermodynamics cycle.

Mechanics, Thermodynamics, and Engineering Economics

15

29

(c) Hint: Carnot Efficiency


Ti To
e=
100%
Ti

Eq.B13

where e = efficiency, Ti = temperature at engine inlet, To = temperature at engine exhaust. Eq.B13


is a measure of the performance of an ideal power cycle or heat engine; it helps to reduce too much
output heat from the low (or cold) temperature. The unit of temperature must be in kelvin.
Given: Ti = 1 500 K (at firing chamber); To = 600 K (at exhaust chamber)
Required: e
1 500 600
e = 100% = 60%
1 500

16

(d) Hint: Ideal-Gas Equation


pV = nRT

Eq.B14

where p = pressure, V = volume, n = number of moles, R = proportionality constant (or gas constant),
T = temperature. Eq.B14 is formed according to the measurements of the behavior of various gases:
1) The volume is proportional to the number of moles.
2) The volume varies inversely with the absolute pressure.
3) The pressure is proportional to the absolute temperature.
Given: STP; V = 2.5 L
Required: n
STP (or standard temperature and pressure) is widely used as a standard point for expression of the
properties and processes of ideal gas. The standard temperature is the freezing point of water, and
the standard pressure is one atmosphere. These can be quantified as follows:

Standard temperature: 0C = 273.15 K

Standard pressure: 1 atm = 760 mmHg = 101.3 kPa

Standard volume of 1 mole of an ideal at STP: 22.4 L
The gas constant was calculated and has different values with appropriate units:
J
L atm
R = 8.314 and R = 0.08206
mol K
mol K
Solving n yields:
or

(101.3 103 Pa)(0.0025 m3) = n(8.314 J/mol K)(273.15 K)


n = 0.1115 mol = 6.7 1022

(1 atm)(2.5 L) = n(0.08206 L atm/mol K)(273.15 K)


n = 0.1115 mol = 6.7 1022
Here are the conversions below.
Liter to cubic meter: 1 L = 1 103 m3; mole to the number of molecules: 1 mol = 6.022 1023

17

(c) Hint: Ideal-Gas in Constant Mass


p1V1 p2V2
=
T1
T2

Eq.B15

From Eq.B14 for a constant mass (or constant n) of an ideal gas the product nR is constant, the

30

Mechanics, Thermodynamics, and Engineering Economics


quantity pV/T is also constant. If the subscripts 1 and 2 refer to any two states of the same mass of
a gas, then Eq.B15 forms.
Given: p1 = p; V1 = V; T1 = T; V2 = 4V; T2 = 2T
Let the first state be a multiple to the second state.
Required: p2
pV (p2)(4V)
=
T
2T
p2 = p/2

18

(d) Hint: Absolute Pressure and Gauge Pressure


pabs = patm + pg

Eq.B16

where pabs = absolute pressure, patm = atmospheric pressure, pg = gauge pressure. Eq.16 describes
an excess pressure above atmospheric pressure. The gauge pressure is important such as an inflated
tire since a pressure of flat tire is an atmospheric pressure.
Given: patm = 755 mmHg; pg = 205 kPa or 1 537.5 mmHg
Take note that 1 kPa = 7.5 mmHg.
Required: pabs
pabs = 755 + 1 537.5 = 2 292.5 mmHg or 2.29 mHg

19
20

(a) Charles Law states that the volume of a gas under constant pressure increases or decreases
with temperature.
(b) Hint: Ideal-Gas in Constant Temperature
Eq.B17

p1V1 = p2V2
If the temperature is constant in Eq.B15, then Eq.B17 forms and refers to Boyles Law.
Given: p1 = 1 atm; V1 = 2 000 mL; p2 = 5 atm
Required: V2
1(2 000) = 5V2
V2 = 400 mL

21

(b) Hint: Celsius to Fahrenheit and vice versa


F = [(9/5)C] + 32

and

C = (F 32)(5/9)

Eqs.B18 & B19

Eq.B18 is used to find the value of Fahrenheit, and Eq.B19 is for Celsius.
Set F = C to solve.

C = [(9/5)C] + 32
C = 40
or
F = (F 32)(5/9)
F = 40

22

(d) Perpetual motion violates the laws of thermodynamics but in most cases, it violates specifically more
on the second law of thermodynamics on which a kind of perpetual motion requires a spontaneous
heat into mechanical work without any side effects (such as cold temperature or surroundings).

Mechanics, Thermodynamics, and Engineering Economics

23

31

(c) Hint: Ideal-Gas in Constant Mass (See Eq.B15)


Given: Use STP for first states pressure and temperature. See STP on Item 16.
p1 = 101.3 kPa; V1 = 1 000 mL; T1 = 273.15 K; p2 = 2p1 (p1 is doubled in the second state);
T2 = 298.15 K
Required: V2
p1(1 000) (2p2)V2
=
273.15
298.15
V2 = 546 mL
It is not necessary to convert L to m3 because the final unit must be in L.

24
25
26

(b) Interest is the payment for the used of borrowed money.


(a) Perpetuity is the type of annuity where payments continue infinitely.
(a) Hint: Future Worth from Simple Interest
Eq.B20

F = P + I = P(1 + ni)

where F = future worth, P = present worth (or principal), I = simple interest, n = number of periods,
i = interest rate.
Such a basic principle when a person borrows a money from an investor, the present money will be
calculated with interest for the future over the period of time he would pay the due.
For simple interest (I = Pni), the present worth is only calculated with ignoring any interest accrued in
preceding interest periods; this is added to the original present worth for the future worth (Eq.B20).
Given: P = 2 000; n = 105 days or 0.288 years; i = 0.12
Required: F
F = 2 000 [1 + (0.288)(0.12)] 2 070

27

(d) Hint: Future Worth from Compound Interest or F/P Equation


Eq.B21

F = P(1 + i)n

For compound interest, the interest accrued for each interest period is calculated on the principal
plus the total amount of interest accumulated in all previous periods. Thus compound interest means
interest on top of interest.
The formula of compound interest is not shown. However, for instance, it is applied to derive Eq.B21
and easy to memorize. Eq.B21 is also called F/P equation, meaning the future worth is unknown,
and the present worth and factor are given. In Eq.B21, the factor is (1 + i)n and called F/P factor.
Given: P = 20 000; n = 17 years; i = 0.065
Required: F
F = 20 000(1 + 0.065)17 = 58 340.93

28

(a) Hint: Effective Rate Interest (ERI)


r
ERI = effective i per time period = 1 +
m

Eq.B22

32

Mechanics, Thermodynamics, and Engineering Economics


where i = effective rate for specified time period, r = nominal interest rate for same time period,
m = number of times interest is compounded per stated time period.
ERI deals when cash flows involve frequencies and compounding periods other than one year with
the nominal rate r. The nominal rate r/m is actually the interest per a year.
Say, the nominal rate is 12% monthly, so the ERI is 12.68%. Thus, additional interest on interest
results in an annual effective return that is larger than 12%. The ERI is almost given as an annual
rate at which it provides a means of comparing interest rates of different compounding frequencies.
Given: r1 = 0.25; m1 = 4 (compounded quarterly); m2 = 2 (compounded semiannually)
Required: r2
In this problem, there are two different nominal rates with equal ERI. So equate the two expressions.
r
1 + 1
m1

m1

r
1 = 1 + 2
m2
4
r
0.25
1 + = 1 + 2
4
2

m2

1
2

r22
1.2744 = 1 + r2 +
4
5.0976 = 4 + 4r2 + r22
0 = 1.0976 + 4r2 + r22
Apply quadratic formula and choose a positive integer.
r2 = 0.2578 = 25.78%

29

(d) Hint: P/F Equation


Comparing Eq.B21, the present worth is unknown in this condition.
Given: F = 290 000; n = 4 years; i = 0.09
Required: P
290 000
F
P = n = 4 = 205 443.31
(1 + i) (1 + 0.09)

30

(b) Hint: F/P Equation (See Eq.B21)


Given: Fig.B4 shows a cash flow diagram of the problem.
Required: the new future worth F after the first future worth F1 was partially withdrawn
F=?
i = 15%
0
P1 = 15 000

F1

i = 15%
16

year

P = F1 P1
Fig.B4

Cash flow digram is a graphical representation on cash flows drawn on the y-axis with a time scale

Mechanics, Thermodynamics, and Engineering Economics

33

on the x-axis. The present worth is drawn in arrow down to determine that its value is lower than
the future worth as the cash flow increases at the certain time.
Analyze Fig.B4. 15 000 was deposited at the beginning. After eight years, the savings has increased;
however it was withdrawn with 15 000. The deducted savings which is considered as a new present
worth P continues to raise within another 8 years with the same interest. Solving P directly yields:
P = F1 P1 = P1(1 + i)n P1 = P1[(1 + i)n 1]

= 15 000[(1 + 0.15)8 1] = 30 885.34
Finally the new future worth is:
F = P(1 + i)n = 30 885.34(1 + 0.15)8 = 94 478.97

31

(d) Hint: F/P Equation within Term


r
F=P 1+
m

mn

Eq.B23

where n = term; mn represents the total number of compounding periods.


Term means the number of years over which the compounding takes place. Unlike compounding
frequency, the number of compounding periods occurs in a year; hence, the impact of term length
is not subject to the law of diminishing returns.
If you still refer on compounding frequency, just ignore the term n as F = P(1 + r/m)m.
Given: P; F = 4P (future worth is four times the present worth); m = 1 (compounded annually);

r/m = 10%/1 or 0.10 (nominal rate)
Required: n
4P = P(1 + 0.10)(1)n
4 = (1.10)n
ln 4 = n(ln 1.10)
n = 14.55 15 years
At the third step of our solution, the natural logarithm is taken on both sides to drop the exponent.
For the final answer, estimate the time higher to sustain much higher future worth.

32

(b) Hint: P/A Equation


(1 + i)n 1
P=A
i(1 + i)n

Eq.B24

The annual worth A is a series maintaining equal end-of-period cash flows (or investments). An
expression for the present worth can be determined by considering each A value as future worth F
until it simplifies as shown in Eq.B24.
Given: A = 3 000; n = 3 years; i = 0.08
Required: P
(1 + 0.08)3 1
P = 3 000 3 = 7 731.29
0.08(1 + 0.08)

33

(a) Hint: Compounded Continuously


F = Peni

Eq.B25

Eq.B25 is used when the compounding period is very small (such as hours, minutes and seconds), or

34

Mechanics, Thermodynamics, and Engineering Economics


infinite.
Given: P = 2340; i = 0.031; n = 3
Required: F
F = 2 340 e3(0.031) = 2 568.06

34
35

(d) Annuity is the series of equal payments made at the equal intervals of time.
(b) Hint: A/F Equation
i
A=F
(1 + i)n 1

Eq.B26

Eq.B26 is used to determine sinking funds. To derive Eq.B26, the P/F equation is substituted into
P/A equation (from Eq.B24).
Given: F = 20 000; n = 12 years; i = 0.06
Required: A
0.06
A = 20 000
= 1 185.54
(1 + 0.06)12 1

36

(b) Hint: F/P Equation within Term (See Eq.B23) and ERI (See Eq.B22)
Given: P = 170 000; F = 185 000 n = 3 years; m = 1 (compounded annually)
Required: ERI
Solve r first.

185 000 = 170 000(1 + r/1)1(3)


r = 0.02859
Then the ERI is:

ERI = (1 + 0.02859)1 1 = 0.02859 = 2.859%

C
1
2
3
4
5
6
7
8
9
10

Differential Equation and


Advanced Mathematics

35

Pretest and Post-test Items

Newtons law of cooling states that the time of change in temperature of an object varies as the
difference in temperature between the object and surroundings. If an object cools from 100C and
50C in 10 minutes, what is the temperature of the object in 5 minutes if the surrounding temperature
is 30C?
a. 54.4C
c. 64.4C
b. 67.4C
d. 50.4C
Solve (2x + 10y3) dy/dx + y = 0.
a. x + y2 = 10y + C
b. xy2 = 10y + C

c. xy2 = (10y5/5) + C
d. x y2 = (10y5/5) + C

If the inverse of a matrix is zero, then it is called _____.


a. antisymmetric matrix
c. singular matrix
b. plural matrix
d. symmetric matrix
Evaluate
a. 2e
b. 2

dx .

+k

x 1 e

x e

x +1 e

c. 2 1
+k

d. 2

+k
+k

The equation (3xy2 + 2y) dx + (2x2y + x) dy = 0 is _____.


a. an exact DE
c. a variable separable
b. a linear DE
d. not an exact
If the right side of a linear DE is zero, then the equation is _____.
a. exact
c. homogeneous
b. All of the above
d. nonhomogeneous
Determine the order of the differential equation d3y/dx3 + 5d2y/dx2 + 2y = e3x.
a. 1
c. 4
b. 3
d. 2
In the equation dz/dr + 2rz = 10, z is _____.
a. a random variable
b. an independent variable

c. a dependent variable
d. a real variable

The equation dy/dx + x/y = 0 represents a family of _____.


a. hyperbolas
c. circles
b. curves
d. parabolas
Which of the following represents an Ordinary Linear DE?

36

11
12
13
14
15
16
17
18
19

Differential Equation and Advanced Mathematics


a. d2y/dx2 + 2 dy/dx + 3y = 0
b. d2y/dx2 + 2 dy/dx + 3y = y3

c. d2y/dx2 + 2 dy/dx + 3xy = 0


d. d2y/dx2 + 2y dy/dx + 3y = 0

11
Evaluate 0
22
a. 0
b. 10

c. 0.32
d. 2.2

22
0
32

32
0 .
11

What is the Laplace transform of e2t sin (10t) u(t)?


s2
c.
a.
(s 2)2 + 100
10
b.
d.
(s 2)2 + 100

s+2

(s + 2)2 + 100
10

(s + 2)2 + 100

A radioactive material decays at a rate proportional to its amount. If the material has a half-life of
30 minutes, what percent of the original mass is expected to remain after 0.8 hours?
a. 22%
c. 28%
b. 38%
d. 33%
A certain bacteria culture is known to grow at a rate proportional to the number of bacteria present.
After one hour, 1 000 strands of bacteria are seen in the culture. After three hours, this number has
tripled. Find the number of strands of bacteria originally in the culture?
a. 688
c. 694
b. 646
d. 628
An object is thrown vertically upward from the ground with initial velocity of 19.6 m/s. Neglecting
air resistance, find the time when maximum height is reached.
a. 4 minutes
c. 4 seconds
b. 2 seconds
d. 2 minutes
If d2y(x)/dx2 + 5 dy(x)/dx + 6y(x) = 2x2 + 2 sin x, its trial solution is _____.
c. Ae2x + Be3x
a. ax2 + bx + c + A sin x + B cos x
2
b. ax + A sin x
d. ax2 + A cos x
The type of series where the operating point is zero.
a. Fourier series
c. Power series
b. Taylor series
d. Maclaurin series
Which of the following satisfies d2y/dx2 y = 0?
a. Aex + Be2x
c. Aex + Bxex
x
x
b. Axe + Be
d. Aex + Bex
Which of the following is the Maclaurin series expansion of sin x?
a. x x3/6 + x5/120 x7/4 048 +
c. 1 x2/2 + x4/24 x6/780 +
b. x x3/6 + x5/120 x7/5 040 +
d. x x2/2 + x4/24 x6/720 +

Differential Equation and Advanced Mathematics

20

37

A container holds 200 gallons of brine solution containing 50 lbs of salt. Initially, a freshwater is
poured into the tank at a rate of 10 gal/min, while the well stirred mixture leaves the tank at the
same rate. What is the amount of salt in the tank after 5 minutes?
a. 38.94 lbs
c. 36.33 lbs
b. 32.88 lbs
d. 44.51 lbs

38

38

Answers

Solutions with footnotes provide explanations.

(b) Hint: Variable Separable


It is the most basic solution in the first-order differential equation (DE). To solve the equation:
dy g(x)
=
dx h(y)
1) Arrange the functions by multiplying dx and dy accordingly.
h(y) dy = g(x) dx
2) Then integrate both sides.
h(y) dy =

g(x) dx

H(y) = G(x) + C
where C is called a constant of integration.
Before identifying the given and requirement, take the Newtons law of cooling into an equation.
dT
= k(T Ts)
Eq.C1a
dt
where dT/dt = rate of change of temperature, k = proportionally constant, T Ts = temperature of
an object that affects the temperature of the surroundings.
Apply variable separable to solve for T.

a)
b)
c)
d)

dT
= k dt
T Ts
dT
= k dt
T Ts
ln |T Ts| = kt + K
eln |T Ts| = ekt + K
T Ts = ekt eK = Cekt
T = Cekt + Ts

Eq.C1b

a) ln |T Ts| is based on the integral of (1/x) dx = ln |x|. For the constant of integration, let it as
K because the solution is not yet finalized; that is called an implicit general solution.
b) To drop |T Ts| and cancel the natural logarithm, exponentiate on both sides.
c) Substitute eK into C since both are constant.
d) That is called an explicit general solution.
Given: T at initial time = 100C; T at 10 minutes = 50C; Ts = 30C
Based on the function y(x) or T(t), you can simplify the term as T(0) = 100C, and T(10) = 50C.
Required: T in 5 minutes, or T(5)
Since there are initial conditions in a domain, the initial value problem (IVP) is introduced. Then
the value of the constant of integration can be solved. If there is a second IVP, there is unknown
constant of that general solution.

Differential Equation and Advanced Mathematics

39

Solving C where the IVP is T(0) = 100C yields:


100 = Cek(0) + 30
100 = C + 30
C = 70
Solving k where the second IVP is T(10) = 50C yields:
50 = 70ek(10) + 30
e10k = 20/70
10k = ln |20/70|
k = 0.125
Eq.C1b becomes:
T = 70 e0.125t + 30
This is called a particular solution.
At t = 5 minutes,

T(5) = 70 e0.125(5) + 30 = 67.4C

(c) Hint: Linear First-Order Equation


A linear first-order equation is the another solution of DE if the DE cannot be able to separate the
variables. However, explicit solution might be impossible. The steps are as follows:
1) Write the equation in the standard form:
dy
+ P(x)y = Q(x)
dx
2) Calculate the integrating factor (x) by the formula:
(x) = e

P(x) dx

Eq.C2a
Eq.C2b

3) Multiply the equation in standard form by (x) and, yielding that the left-hand side is just
d/dx [(x)y], obtain
dy
(x) + P(x)(x)y = (x)Q(x)
dx
d
[(x)y]
dx

= (x)Q(x)

Eq.C2c

That left-hand side is shortened because it is a derivative. If you want to verify d/dx [(x)y],
use the product rule.
4) Integrate the last equation.
Given: (2x + 10y3) dy/dx + y = 0
Required: the implicit solution
Take the standard form by following Eq.C2a.
1
dy
+ 3 y = 0
2x + 10y
dx
This case cannot proceed to the succeeding steps because P(x) has the presence of y3. The best way
to do is to exchange between the independent and dependent variable in order to be a real linear
equation. So multiply dx/dy both sides and arrange the form.

40

Differential Equation and Advanced Mathematics


dy
dx
(2x + 10y3) + y = 0
dx
dy
dx
(2x + 10y3) + y = 0
dy
dx 2x + 10y3
+=0
dy
y
dx 2
+ x = 10y2
dy y
The dependent variable is now x, and Eq.C2a inverses into dx/dy + P(y)x = Q(y). The integrating
factor (y) is:
(y) = e (2/y) dy = e2 ln |y| = eln |y2| = y2
Multiply (y) both sides until the implicit solution exists.
dx 2x
+ = 10y2 y2
dy y
dx
y2 + 2xy = 10y4
dy
d
(xy2) = 10y4
dy
a)

d(xy2) =

10y4 dy

10y5
xy2 = + C
5
a) Integrating without dx or dy is neglected.

3
4

(c) If the inverse of a matrix is zero, then it is called singular matrix.


(b) Hint: Integration by Substitution and Integration by Parts

udv = uv

vdu

Eq.C3

An integration by substitution is a method for algebraically simplifying the form of a function so


that its integral can be easily recognized. On the other hand, an integration by parts is a theorem
that relates the integral having a product of functions, and the standard form shows on Eq.C3. Some
functions in an integral can be solved both or any one of these techniques.
Given:

dx

Since this is not a product of functions, use the integration


by substitution.
Select a function that

(1/2)
can be possibly simplified in that integral, which is x . Let w = x or x . The dx Requireds also
to be substituted.
[w = x(1/2)] d/dx
dw = (1/2)x (1/2) dx
dx = 2w dw

Differential Equation and Advanced Mathematics

41

The substituted function in that integral becomes:


2wew dw
A product of function exists. Use the integration by parts by letting:
u=w
a)
du = dw
and
dv = ew dw
b)
v = ew
Following Eq.C3 becomes:
c)

2wew dw = 2 wew

ew dw

= 2(wew ew) + k = 2(w 1)ew + k


Substitute back the original parts.

x 1 e

+k

a) The u = w is differentiated to obtain du = dw.


b) The dv = ew dw is integrated to obtain v = ew.
c) A function with constant in an integral is always factored out.

(d) Given: (3xy2 + 2y) dx + (2x2y + x) dy = 0


Required: possible methods
Based on the choices, attempt these three methods: variable separable, linear equation, and exact
equation. Since you are guided on how to solve variable separable and linear equation, none of these
methods are not reliably applied, and here are the reasons:
Not a variable separable: All x and y cannot be grouped with respect to dx and dy.
Not a linear equation: By using Eq.C2a, P(x) consists a lot of y variables. Exchanging independent
and dependent variables, which is related on the solution of Item 2, do not work effectively as well.
The last method as our expectation is the exact equation. This process is to verify the two functions
if exact by exchanging and partially differentiating with respect to dx and dy. So,

(3xy2 + 2y) = 6xy + 2 and


(2x2y + x) = 4xy + 1
y
x
A variable with the respect to other differential variable is considered a constant.
Unfortunately, 6xy + 3 4xy + 1 which implies that the given is not an exact equation. There is
a possible solution for that equation called an integrating factor of non-exact equation. It helps the
non-exact equation to be exact.

6
7
8

(c) If the right-hand side of a linear DE is zero, the equation is homogeneous.


(b) The higher order of d3y/dx3 + 5d2y/dx2 + 2y = e3x is 3 because of d3y/dx3.
(c) In the equation dz/dr + 2rz = 10, z is a dependent variable.

42

Differential Equation and Advanced Mathematics

9
10

(c) The equation dy/dx + x/y = 0 represents a family of circles.


To proof this statement, apply variable separable and you will get x2 + y2 = 2C which relates to the

equation of a circle: x2 + y2 = r2. Following that form, where r2 = 2C or r = 2C , becomes x2 + y2 =


2
2C .
(a) Given: From the choices:

a. d2y/dx2 + 2 dy/dx + 3y = 0

b. d2y/dx2 + 2 dy/dx + 3y = y3

c. d2y/dx2 + 2 dy/dx + 3xy = 0

d. d2y/dx2 + 2y dy/dx + 3y = 0
Required: One of them represents a linear ordinary differential equation (ODE).
ODE can be either linear or nonlinear. To verify that ODE is a linear if:
1) It involves ordinary n-order derivatives.
2) No presence of dependent variables be a product of (1); otherwise, it is a nonlinear DE.
3) No presence of dependent variables with a power be a product of (1), or in a term, in which it
can also be a nonlinear. A single dependent variable in a term is a linear.
For the correct answer, d2y/dx2 + 2 dy/dx + 3y = 0 shows the linear ODE because there are no
signs of dependent variable with each derivative, and there is a one dependent variable in a term,
that is 3y.

11

(a) Hint: The Solution of 3 by 3 Determinant


There are different styles of solving a determinant, and just take this one.
Extend the determinants grid by putting again the first two columns of elements.
a1
a2
a3

b1
b2
b3

c1
c2
c3

a1
a2
a3

b1
b2
b3

c1
c2
c3

a1
a2
a3

b1
b2
b3

Then multiply along the down-diagonals and along the up-diagonals. Add the down-diagonals and
subtract the up-diagonals.
a1
a2
a3

b1
b2
b3

c1
c2
c3

a1
a2
a3

b1
b2
b3

= a1b2c3 + b1c2a3 + c1a2b3 a3b2c1 b3c2a1 c3a2b1


11
Given: 0
22

22
0
32

Eq.C4

32
0
11

Notice that each diagonal is multiplied always results zero. Automatically, the answer is zero.

12

(d) Hint: Familiarizing Laplace Transform of the Functions


Given: e2t sin (10t) u(t)
Using the formula of a Laplace transform takes much time to solve many useful functions. A guide

Differential Equation and Advanced Mathematics

43

which is a table of Laplace transform is better to be memorized the functions. Based on the given,
Table 1 is provided.
Time domain
f(t)
u(t)

Frequency domain
F(s)
1
1

s+a

eat
sin t

2
s + 2

eat sin t

2
(s + a)2 + 2
Table 1

Therefore,

13

10
{e2t sin (10t) u(t)} =
(s + 2)2 + 100

(d) Hint: Variable Separable


This problem is actually the same concept with Item 1. Take the first statement into an equation.
dA/dt = kA
where dA/dt is a rate of change of the amount.
Applying variable separable yields:

A = C ekt

Eq.C5

Given: A(0) = 100% (the amount of the material is full at initial); A(30) = 50% (its amount is half
at 30 minutes)
Required: A at t = 0.8 hours or 48 minutes
Solving C yields:
Solving k yields:

Eq.C5 becomes:
At t = 48,

14

1 = C ek(0)
C=1
0.5 = (1) ek(30)
k = 0.023105
A = e0.023105t
A(48) = e0.023105(48) = 0.33 = 33%

(c) Hint: Variable Separable


This problem is even the same concept with Items 1 and 13, however, there is a difference regarding
of the given. Take the first statement into an equation and obtain immediately the general solution.
dP/dt = kP
P = Cekt

Eq.C6

44

Differential Equation and Advanced Mathematics


where dP/dt is a rate of change of the population.
Given: P(1) = 1 000 (there are 1000 bacteria at one hour); P(4) = 3 000 (after three hours with the
total of 4 hours, the number of bacteria multiplies in 3 000)
Required: initial population of the bacteria
From the given, Eq.C6 becomes:
1 000 = C ek

and

3 000 = C e4k

Since their constant of integrations are equal, equate the two solutions above to solve k.
C = 1 000 ek = 3 000 e4k
k 4k
e e = 3 000/1 000
ln [e3k = 3]
ln 3
k = = 0.3662
3
k
From 1 000 = C e , the value of C is:
C = 1 000 e0.3662 = 639.36
Eq.C6 finally becomes:
At t = 0 for initial population,

15

P = 693.36 e0.3662t
P(0) = 693.36 e0.3662(0) 694

(b) Hint: Free Fall at the Rate of Change


Here, the acceleration becomes the rate of change. Since it is not a constant, the Newtons second
law is introduced where the force is a product of mass and acceleration. In free fall due to the rate
of change, the equation is defined as:
dv
m = mg
dt
where m = mass, dv/dt = rate of change of the acceleration, g = acceleration of gravity.
Applying variable separable yields:

v = gt + C

Eq.C7

where v is a velocity.
Given: t = 0; v = 19.6 m/s; g = 9.8 m/s2 (constant)
Required: t at maximum height
Solving C at t = 0 yields

19.6 = 9.8(0) + C
C = 19.6
Eq.C7 becomes:

v = 9.8t + 19.6

When the object reaches its maximum height, the velocity is zero, and solving t yields:
0 = 9.8t + 19.6
t = 2 s

16

(a) Hint: Trial Solution of a Nonhomogeneous Equation


The trial solution is a sophisticated way to solve nonhomogeneous equation in which the right-hand

Differential Equation and Advanced Mathematics

45

side of the equation is employed of guessing with the variety of standard forms.
Given: d2y(x)/dx2 + 5 dy(x)/dx + 6y(x) = 2x2 + 2 sin x,
Required: its trial solution
The first procedure in the trial solution is called a particular solution yp(x). Here, there are two terms:
2x2 and 2 sin x.
Guessing 2x2 in a standard form with coefficients are as follows in different degrees of polynomial:
ax + b
2
ax + bx + c
ax3 + bx2 + cx + d
and so on. Thus, it suits with ax2 + bx + c because of the power (or degree) which is 2.
Guessing trigonometric functions like sin x would be A sin x but this will fail because the derivatives
introduce cosine terms:
A sin x + B cos x
Add the particular solution.

17

yp(x) = ax2 + bx + c + A sin x + B cos x

(d) The answer is Maclaurin Series and the formula is:

f (0)
f n(0)
f (0)
xn = f(0) + x + x2 +
2!
n!
1!
n=0

Eq.C8

As to expand the summation, the continuous derivative of a function: f(0), f (0), f (0), , f n(0)
takes the independent variable equal to zero.

18

(b) Hint: Complementary Solution

yc(x) = A em1x + B em2x

where m1 and m2 are roots.


If the particular solution yp(x) is determined in the trial solution the complementary solution yc(x)
is next. To solve complementary solution, let the right-hand side of the equation zero. It is derived
from the second-order DE and turned into Eq.C9.
Given: d2y/dx2 y = 0
Required: its complementary solution
The derivatives can be expressed from Leibnizs notation to Lagranges notation such that d2y/dx2 =
y and dy/dx = y. Let y = D2 and y = 1 to be such an algebraic form and then solve the roots.
y 1 = 0
D2 1 = 0
(D + 1)(D 1) = 0
where D is a del operator. The roots are D1 = 1 and D2 = 1. Following Eq.C9 becomes:
yc(x) = A ex + B ex

19

(b) Hint: Maclaurin Series (See Eq.8)


List the increasing order of derivates of sin x as possible and evaluate them where x = 0.
f(x) = sin x f (x) = cos x f (x) = sin x f (x) = cos x
f(0) = 0
f (0) = 1
f (0) = 0
f (x) = 1

Eq.C9

46

Differential Equation and Advanced Mathematics


Notice that in the fourth derivative, the function returns back from the original, so f(x) = f 4(x) = sin x.
Following Eq.C8 becomes:
x
x3
x5
x7
sin x = 0 + + 0 + 0 + + 0 +
3!
5!
1!
7!
3
5
7
x
x
x
=x++
6 120 5 040

20

(a) Hint: Variable Separable (Same concept of the rest of word problems using this method)
This word problem is a compartment analysis involving mixing problems. Equating a basic onecompartment system suggests
dQ/dt = input rate output rate
Eq.C10
where Q is the amount of the specific material.
Given: Fig.C1 shows the problem (or model).
Required: the output rate of salt Q leaving the tank in 5 minutes
freshwater
10 gal/min

a tank with 200 gallons


of brine

Q(t)
Q(0) = 50 lbs

Q(5 min) = ?
10 gal/min

Fig.C1
Determine the amount of salt in the freshwater and the brine. Of course, the freshwater contains no
salt while the brine contains 50 lbs of salt as given. So calculating the input and output rate will be:
(10 gal/min)(0 lbs/gal) = 0 lbs/min
and
Q
(10 gal/min) lbs/gal = 0.05Q lbs/min
200
Following Eq.C10 becomes:

dQ/dt = 0.05Q

Applying variable separable for the general equation yields:


Q = C e0.05t
There is only one condition here. Solving C where Q(0) = 50 yields:
50 = C e0.05(0)
C = 50
The general equation above becomes:
At t = 5,

Q = 50 e0.05t

Q(5) = 50 e0.05(5) = 38.94 lbs

47

Quick Tips

Another encountered session that we are going to calculate. There are some unique techniques that will make
your step convenient. Good if you have been familiarized the keys of the calculator.

Exponential Function of a Differential Equation


Word problems involving Newtons law of cooling, mixing problems, material decay, etc. are commonly the
exponential function. You dont need to solve manually the differential equation (DE) with that case just to
get the general solution. We will distinguish techniques which depend the form of the exponential function.
However this is only applicable for a variable separable.
Basic Exponential Function of a DE
We follow the format of x = C ekt where x = dependent variable, C = constant of integration, k = proportionally
constant, and t = time. This relates with the material decay, and rate of population (or growing microorganisms).
If x = C ekt, the calculator does not matter if k is negative or positive as long as the two conditions (known as
initial value problems, IVPs) are given.
1) Lets choose in Item 13. Recalling two IVPs that A = 100% at t = 0, and A = 50% at t = 30 minutes where
A (which is the dependent variable) is the amount of the material. Obtain A at t = 48 minutes.
We can calculate this in a statistical way. On the calculator, press w35 for the exponential function.

2) The x-column is for time t (which is the independent variable) and the y-column is for A. Input them as
the screen below is shown.

3) Press C and go to q155 for y-regression.

48

Differential Equation and Advanced Mathematics


Caution: Do not press W because everything that you have inputted will be lost. If happens, then you
input again.
You will see that symbol alone and type 48 before it. Then press p.

An Exponential Function as x x0 = C ekt


This format is very related with the Newtons law of cooling: T Ts = C ekt where T Ts = temperature of an
object that affects the temperature of the surroundings.
1) Lets choose in Item 1. Recalling two IVPs and Ts that T = 100C at t = 0, T = 50C at t = 10 minutes,
and Ts = 30C. Obtain T at t = 5 minutes.
Same process, press w35 for the exponential function. Make sure you will subtract Ts from T, and
these will be 70C and 20C, respectively. Input them as the screen below is shown.

2) Press C and go to q155 for y-regression. Type 5 before y-regression followed by +3 0


p.

We need to add Ts = 30 with that case because the two IVPs have been subtracted by 30. So you have to
recover the value of T by Ts at any time t.
Note: This technique is not same with heating in DE where the equation is T + Ts = C ekt. However you can
manipulate it in the calculator, and one of it is to change from the operation of subtraction to addition.

Determinant
In a matrix function of the calculator, we can calculate a determinant. Of course, the determinant must be in
the form of square of elements such as 2 2, and 3 3. Lets consider a new example of 3 3 determinant
1
3
3

2
2
5

1
4
6

1) Press w6 for MATRIX. It asks what the item of matrix you are going to select. Lets work with
Matrix A, so press 1. It asks again by how many elements; since we are dealing about 3 by 3 matrix,
press 1. After that, you see 9 zeroes in Matrix A.

Differential Equation and Advanced Mathematics

49

2) Input all the elements.

3) Press C to exit. Press q1 for MATRIX controls. Press 7 for determinant. Press again q1
and 3 for your inputted Matrix A. Lastly, press p.

The Reverse Engineering on Indefinite Integrals


Doing a reverse engineering is not stupid at all, but we intentionally manipulate shortcut and wise solution on
which we are basing the choices. For some ways, it can waste your time.
If you dont know how to evaluate indefinite integral, do the reverse engineering. Lets choose in Item 4, that is
e
and the choices are
(a) 2e

+ k, (b) 2

x 1 e

1) Remove the integral sign of

+ k, (c) 2 1

dx

x e

+ k, and (d) 2

x +1 e

dx . Let x = 0.1. Creating a math format equation, press

+ k.

50

Differential Equation and Advanced Mathematics


qhs0.1p
and we obtain

2) Differentiate the choices for which the opposite of derivative is an integral. Let also x = 0.1. We have
(a) qy2qhsQ)$$$0.1p

(b) qy2(sQ)$-1)qhsQ)$$$0.1p

Notice that (b) has equal output which is approximately equal to 1.37. Therefore, (b) is the answer. We
know that the remaining choices including (a) are clearly failed.
Note: If you evaluate a derivative, the manipulation is just an inversion from our technique using the calculator.

D
1
2
3
4
5
6
7
8
9
10

51

Electromagnetics

Pretest and Post-test Items

If the work done around a closed path is not zero, then the field is a/an _____.
a. non-conservative field
c. conservative field
b. electrically charged
d. electrically neutral
Find the distance between (5, 3/2, 0) and (5, /2, 10) in cylindrical coordinates.

a. 5 10
c. 10 2

b. 2 10
d. 3 10
Find the unit vector directed from (2, 5, 2) toward (14, 5, 3).
a. a = 2a y + a z
c. a = (12/13)a x + (5/13)a y
b. a = (13/12)a x + (5/13)a y
d. a = a x
The ratio /0 is called a/an _____.
a. conductivity
b. absolute permittivity

c. susceptibility
d. relative permittivity

Two identical uniform line charges of L = 4 nC/m are parallel to the z-axis at x = 0, y = 4 m.
Determine the electric field E at (4, 0, z) m.
c. 18a x V/m
a. 10a x V/m
b. 20a x V/m
d. 30a x V/m
Find the distance between (1, /4, 0) and (1, 3/4, ) where the points are given in spherical
coordinates.
a. 3
c. 7
b. 2
d. 5
_____ is the formation of electric dipoles in a material upon exposure to an external electric field.
a. Displacement current
c. Magnetism
b. Induction
d. Polarization
Charge lies on the circular disk r 4 m, z = 0 with the density S = (104/r) C/m2. Determine the
electric field E at r = 0, z = 3 m.
a. 1.51 106 a z V/m
c. 7.82 106 a z V/m
6
b. 2.55 10 a z V/m
d. 6.21 106 a z V/m
Which conductor in the list below has the highest conductivity?
a. copper
c. aluminum
b. gold
d. silver
The value of the permeability of free space 0 is _____.
a. 8.854 1012 F/m
c. 120 rad
8
b. 100 10 H/m
d. 4 107 H/m

52

11
12
13
14
15
16
17
18
19
20

Electromagnetics
How much charge does an electron have?
a. 7 1020 C
b. 1.602 1019 C

c. 6 1019 C
d. 5 1020 C

Given A = 4a y + 10a z and B = 2a x + 3a y, find the projection of A on B.

c. 2
a. 12/ 13

d. 5/ 60
b. 10 2
A directional derivative indicating the max space rate of change of a scalar quantity is _____.
a. Laplacian
c. divergent
b. curl
d. gradient
A point charge Q = 0.4 nC is located at (2, 3, 3) m in Cartesian coordinates. Find the potential
difference VAB where point A is (2, 2, 3) and B is (2, 3, 3) m.
a. 3.50 V
c. 2.99 V
b. 6.70 V
d. 2.70 V
_____ is usually described as circulation per unit area.
a. Induction
c. Curl
b. Gradient
d. Divergence
The value of the vacuum permittivity 0 is _____.
c. 377
a. 10 1020 F/m
12
b. 8.854 10 F/m
d. 6.77 1018 F/m
Find E at (0, 0, 5) m due to Q1 = 0.35 C at (0, 4, 0) and Q2 = 0.55 C at (3, 0, 0) m.
c. 100.2a x 50.9a y 15.2a z V/m
a. 64.8a x 47.9a y 74.8a z V/m
b. 74.8a x 47.9a y 64.8a z V/m
d. 9a x 50.2a y 70.8a z V/m
Lines drawn to indicate the direction of a vector field are _____.
a. contour lines
c. Smith chart
b. coordinate system
d. flux lines
Given A = 2a x + 4a y 3a z and B = a x a y, find the cross product of A and B.
a. 6a x 3a y 3a z
c. 2a x 5a y 7a z
b. 3a x 3a y 6a z
d. 5a x 7a y 2a z
Given A = 2a x + 4a y 3a z and B = a x a y, find the dot product of A and B.
a. 2
c. 7
b. 2
d. 6

Electromagnetics

1
2

53

Answers

(a) If the work done around a closed path is not zero, then the field is a non-conservative.
(b) Hint: Pythagorean Theorem or Formulas of Cylindrical to Rectangular Coordinate Systems
x = cos
y = cos
z=z

Eq.D1

where = radius, = angle of the radius, z = amizuth (or just z-axis) are the cylindrical coordinates.
For instance, there are mathematical techniques to determine the distance between two points in
a coordinate system and one of them is the Pythagorean Theorem. Aside from this theorem, the
conversion of cylindrical to rectangular coordinate system may be suggested as shown in Eq.D1
since the magnitude of distance usually refers on the rectangular coordinates.
Given: (5, 3/2, 0) and (5, /2, 10)
Required: distance d between, say, P1 and P2
Setting up the coordinates of the given points shows:
P1( = 5, = 270 or 90, z = 0) and
Make sure that the radians are changed to degrees.

P2( = 5, = 90, z = 10)

Two ways to solve the distance:


1) Draw the cylindrical coordinate system and place the two points. The point P1 exceeds the angle
from the surface, the solid is imaginarily extended as a revolution (Fig.D1).

P2
z2

1 = 2

d
P1

Fig.D1
Using the Pythagorean Theorem where the two radii and one azimuth are used yields:
d2 = (5 + 5)2 + 102

d = 2 10
2) This solution is not Requiredd to draw as Fig.D1. The coordinates are converted using Eq.D1.
P1(x = 0, y = 5, z = 0) and P2(x = 0, y = 5, z = 10)
Calculating the magnitude instead of vector yields:

P12 = (0 0)2 + [5 (5)]2 + (10 0)2 = 2 10


In vector of P12, subtract from P2 to P1 as shown inside the square root.

54

Electromagnetics

(c) Hint: Unit Vector


A
a=
A
A unit vector is merely a vector in that direction divided by its magnitude.

Eq.D2

Given: (2, 5, 2) toward (14, 5, 3)


Required: the unit vector between the two points
Subtract the vectors.

(14 2)a x + [5 (5)]a y + [3 (2)]a z = 12a x + 5a z

Then,

4
5

5
12a x + 5a z 12
= ax + az
a=

13
122 + 52 13

(d) The ratio /0 is called a relative permittivity.


(c) Hint: Electric Field in a Line Charge

L
E = a
20

Eq.D3

where E = electric field intensity, L = line charge density, 0 = vacuum permittivity, = radial
distance between a portion of the line charge and a field point, a = unit vector of radial distance.
Eq.D3 is used in the concept when the electric field produces from the uniform and infinite line
charge density to a field point (not charge). Note that Eq.D3 represents on the cylindrical coordinate
system because the radial distance is manipulated in terms of cylindrical coordinates.
Given: Fig.D2a shows the problem.
Required: E at (4, 0, z) meters
EB

Pb(4, 0, z)

Eb

Eb
Pb

Pa(4, 0, z)
4

4
+4

+4

+4

L = 4 nC/m
(a) (b)

Pa

+4

Ea

Ea
EA

Fig.D2

Electromagnetics

55

Saying that the field point at (4, 0, z) is PA and second one is PB. Focus first on the subscript a. In
Fig.D2b, the coordinate system faces on the top view to be clearly seen the radial distances and the
electric fields.
Two solutions are as follows:
1) Each radial distance is equal magnitude. Despite the fact that the radial distance does not mean
exclusively for cylindrical coordinates. For this case, let be R, and a be a R to avoid confusion.
Using the Pythagorean Theorem,

R = 42 + (4)2 = 4 2 5.657
Recognize the fact that not all line charges are located along the z-axis. For the two electric
fields Ea, the vectors are taken from the magnitude of R will be 4a x, not 4a x + 4a y and 4a x 4a y
because it matters the location of the field point relative to the line charge. Solve the two electric
fields Ea where 0 = 8.854 1012 farad/meter, F/m.
L
L
L
L
Ea + Ea = a R + a R = a R = 2 R
20R
20R
0R
0R
4 109
=
(4a x) = 18a x V/m
(8.854 1012)(5.6572)
The Eb is equal magnitude with Ea but the direction opposes. Thus,
Eb = 18a x V/m
For the conclusion,

E = 18a x V/m

2) The two electric fields Ea are the vector components of EA because they are intersecting each
other perpendicularly. It becomes a resultant electric field. So
L
R EA
EA =
0R2EA
Subtracting the vectors where the two line charges are at (0, 4, z) and Pa(4, 0, z), will be:
(4 0)a x + [0 (4)]a y = 4a x + 4a y
and
(4 0)a x + (0 4)a y = 4a x 4a y
So the resultant vector and its magnitude of the radial distance of EA are REA = 8a x and REA = 8.
Finally,
4 109
(8a x) = 18a x V/m
EA =
(8.854 1012)(82)
and

EB = 18a x V/m

where REB = 8a x and REB = 8. Then,


E = 18a x V/m

(d) Hint: Possible Math Techniques or Formulas of Spherical to Rectangular Coordinate Systems
x = r sin cos
Eq.D4
y = r sin sin
z = r cos
where the spherical coordinates are r = radius of a sphere, = angle between the z-axis and the line

56

Electromagnetics
drawn from the origin to the point, = angle between the x-axis and the projection in the z = 0
plane of the line drawn from the origin to the point. The angle here is exactly same as the cylindrical
coordinates.
In spherical coordinate system, the Pythagorean Theorem might be inconvenient to solve the distance
depending on the location of the points. Other tools such as analytical geometry or trigonometry
can be provided yet it might take time as to solve manually. Eq.D4 would be suggested.
Given: (1, /4, 0) and (1, 3/4, )
Required: distance between, say, P1 and P2
Setting up the coordinates of the given points shows:
P1(r = 1, = 45, = 0) and P2(r = 1, = 135, = 180)
Make sure that the radians are changed to degrees.
Two ways to solve the distance:
1) Draw the spherical coordinate system and place the two points. The coordinates of point P2
exceeds over the solid, so the curve is imaginarily drawn at the back portion (Fig.D3a). Notice
that the radius of two points meet linearly and form a distance. Simply add two radii:
d = r1 + r2 = 2
Fig.D3b shows the side view from Fig.D3a at which the angles of are opposite each other in
order to be faced a two-dimensional surface.
+z-axis to x-axis
curve
P1

r1

r1
1

P1

r2

P2

x-axis to z-axis
curve
(a) (b)

r2
P2

Fig.D3
2) Use Eq.D4 to convert the coordinates.
P1(x = 0.707, y = 0, z = 0.707) and P2(x = 0.707, y = 0, z = 0.707)

where 0.707 = 2/2. Calculating the magnitude yields:

d = (0.707 0.707)2 + (0 0)2 + (0.707 0.707)2 = 2

7
8

(d) Polarization is the formation of electric dipoles in a material upon exposure to an external
electric field.
(a) Hint: Electric Field in a Surface Charge
Q
E = 2 a R
40R

Eq.D5

Electromagnetics

57

where Q = point charge, R = radial distance. The electric field E gives a force per unit charge. The
force between two charges is stated according to the Coulombs law, and then applied in the definition
of E. If the charge becomes a differential, the E is affected; Eq.D5 will change depending on what
kind of charge density.
Eq.D5 is an electric field in a point charge. This will not be used for the solution. So you are going
to derive another equation because you do not deal with infinite surface charge in this problem.
Given: Reread the problem, or refer directly to Fig.D4.
Required: E at r = 0, z = 3 meters
E

S = (104/r) C/m2

Fig.D4
The solution in Item 5 refers with a line charge density L. Here, the surface charge density S is the
concern. The circular disk in Fig.D4 is not an ellipse which is because of the perspective. Selecting
a piece of area somewhere on the S considers a differential surface charge dS.
Fig.D4 is actually not a spherical coordinate system but a cylindrical because a cylinder has planes.
The tricky part here is the coordinates used for which and r are both called radius. Taking the
differential of the coordinates, the radius r becomes dr, and the other part becomes r d (not d
because it is not a length). Thus,
dS = r d dr
Any charge Q is selected on the dS from the S considers a differential. So,
dQ = S dS = S r d dr
Eq.D5 transforms into:

dQ
dE = 2 a R
40R

Integrating both sides yields:


E=

dQ
2 aR =
40R

S r d dr

R
40R3

Eq.D6

Two integral signs with a ring is called a surface integral. Eq.D6 is now an electric field in a surface
charge. Since Eq.D6 is a definite integral, the limits are 0 r 4, and 0 2 (because the
disk is a circle). The limits of the angle must be in radian. The unit vector a R is the vector of the
field point relative to dS over the magnitude of the radial distance R.

58

Electromagnetics
2

E=

4
0

(104/r) r d dr
3(104)
3 (3a z) = a z

40
r2 + 9
40

2
0

4
0

dr

(r2 + 9) 3/2

Take out the constants and unit vector from the integral at the left. Arrange the functions and limits
with respect to d and dr.
3(104)
= az
40

9 r2 + 9

4
0

3(104)(4/45)

=
az
20

Integrating [1/(r + 9) ] dr can be solved via trigonometric substitution but it takes more time for
you to solve. A calculator having an operation of integral will help.
2

3/2

3(104)(4/45)
=
a z = 1.51 106 a z V/m
12
2(8.854 10 )

9
10
11
12

(d) Silver (6.17 107 siemens/meter, S/m) has the highest conductivity among aluminum (3.82
107 S/m), gold (4.10 107 S/m), and copper (5.80 107 S/m).
(d) The value of the permeability of free space 0 is 4 107 henry/meter, H/m.
(b) The charge of an electron has 1.602 1019 C.
(a) Hint: Dot Product, and Scalar Projection
A B = AB cos = AxBx + AyBy + AzBz

Eq.D7

For multiplying the two vectors that results a scalar quantity (or magnitude), Eq.D7 is used. The
scalar projection will be mentioned on the solution.
Given: A = 4a y + 10a z and B = 2a x + 3a y
Required: the (scalar) projection of A on B
Fig.D5 shows the two-dimensional layout to label clearly the projection of A on B. It forms a right
triangle. The arrows of the vector are connected from the origin O.

A
aA
O

aB

A cos = A aB
Fig.D5
To take the scalar projection of A on B, the dot product will be A and the unit vector of B.
AB AB
A cos = A = = A a B
AB
B

Electromagnetics
Then,

13
14

59

12
A B (4a y + 10a z) (2a x + 3a y)
=
=

B
22 + 32
13

(d) A directional derivative indicating the max space rate of change of a scalar quantity is gradient.
(d) Hint: Potential Field of a Point Charge, and Potential Difference
Q
V = and VAB = VA VB
40R

Eqs.D8 & D9

where V = potential of a charge from one (field) point, VAB = potential difference. Eq.D8 defines the
potential at the radial distance R from a point charge Q at the origin. If a point charge encounters
with two points, a potential difference occurs at which a point charge tends to move (Eq.D9). Both
formulas are scalar.
Given: Q = 0.4 nC at (2, 3, 3) meters; point A at (2, 2, 3); point B at (2, 3, 3) meters
Required: VAB
Determining the radial distances based on the two points, PA and PB, that is, from PA to Q and from
PB to Q, respectively, shows:

RA = (2 2)2 + (2 3)2 + (3 3)2 = 1 and RB = (2 2)2 + (3 3)2 + (3 3)2 = 4


Solving the potential of the two points yields:
0.4 109
VA =
= 3.6 V and
4(8.854 1012)(1)
Then,

15
16
17

0.4 109
VB =
= 0.9 V
4(8.854 1012)(4)

VAB = 2.7 V

(c) Curl is usually described as circulation per unit area.


(b) The value of the vacuum permittivity 0 is 8.854 1012 F/m (or 1/36 109 F/m).
(b) Hint: Electric Field in a Point Charge (See Eq.D5)
Given: Q1 = 0.35 C at (0, 4, 0) meters; Q2 = 0.55 C at (3, 0, 0) meters
Required: E at (0, 0, 5) meters
Solving the vectors and magnitude of the radial distances from E to Q1 and from E to Q2 yields:
R1 = 4a y + 5a z ; R1 6.403 and R2 = 3a x + 5a z ; R2 5.831
Solving the two electric fields yields:
0.35 106
(4a y + 5a z) = 47.9a y + 59.9a z
E1 =
4(8.854 1012)(6.403)3
and
0.55 106
(3a x + 5a z) = 74.8a x 124.7a z
E2 =
4(8.854 1012)(5.831)3
Adding them will be:

60

Electromagnetics
E = 74.8a x 47.9a y 64.8a z

18
19

(d) Lines drawn to indicate the direction of a vector field are flux lines.
(b) Hint: Cross Product
ax
A B = Ax
Bx

ay
Ay
By

az
Az = (AyBz AzBy)a x + (AzBx AxBz)a y + (AxBy AyBx)a z Eq.D10
Bz

For multiplying the two vectors that results a vector quantity, Eq.D10 is used. That expanded form
can be found out through a determinant. This is not a commutative law.
Given: A = 2a x + 4a y 3a z and B = a x a y
Required: A B
ax
2
1

20

ay
4
1

az
3 = (0 3)a x + (3 0)a y + (2 4)a z = 3a x 3a y 6a z
0

(b) Hint: Dot Product (See Eq.D7)


Given: A = 2a x + 4a y 3a z and B = a x a y
Required: A B
2(1) + 4(1) + (3)(0) = 2

Electromagnetics

61

Quick Tips

This is the last section of Quick Tips. We will encounter the vector and scalar calculations. Here, the techniques
are simply shortened the solution.

Dot and Cross Product


To solve the dot and cross product, you dont need to apply those formulas. Instead use the calculator. Lets
choose in Items 19 where the given two vectors are
A = 2a x + 4a y 3a z and B = a x a y
Dot Product
1) Press w8 for VECTOR. It asks what the item of vector you are going to select; for the first vector, so
press 1 for Vector A. It asks again what number of coordinates will be; so press 1. After that, you see
three zeroes in Vector A.

2) Input the coordinates for Vector A.

For Vector B, directly press q5221

62

Electromagnetics
and input the coordinates.

Caution: For the second vector, do not press w after inputting the first vector because your data will
be lost. Thats why you should go to q52 (or go to Cq221).
3) Press C. Then for dot product, simply follow these consecutive keys:
q53q57q54p

Cross Product
With the same input vectors A and B, simply follow these consecutive keys:
q53Oq54p

Other Applications of Scalar and Vector


Scalar Projection
Lets choose in Item 12, with the following given A = 4a y + 10a z and B = 2a x + 3a y. We require to get the
scalar projection of A on B. Make sure you are now familiar the keys and controls of VECTOR.
1) For vectors A and B, we input their coordinates such that

2) We know that the scalar projection of A on B is


AB

B
However, you cannot create a math format equation in VECTOR. Change it into line format equation as
(A B) B
Do not calculate the magnitude of Vector B; instead use the absolute function by pressing qc. Follow
these consecutive keys:

Electromagnetics

63

(q53q57q54)Pqcq54p

Other Vector Calculations such as Electric Field


Lets choose in Item 17 with the expression
0.35 106
E =
R
40R3
where R = (4a y + 5a z).
1) Changing it into a line format equation, we have
[0.35 106 (40R3)] R
Before inputting it to the calculator, we necessarily put a cross product sign because the electric field is a
vector quantity.
Constants such as the ratio of a circles circumference to the diameter (which is known as ) and the vacuum
permittivity 0 can be found in the calculator. For , the keys are qK.
The back of the calculators cover case has a sticker where you can see the list of the scientific constants.
So the vacuum permittivity 0 is located at number 32; the keys are q7 (CONST) followed by 32.
2) On the calculator, let R as Vector A. Then input the coordinates.

3) For the expression, follow these consecutive keys:


(0.35K-6P(4qKq732
qcq53)qd)Oq53p

64

1
2
3
4
5
6
7
8

9
10

Electrical Circuits and DC Machines


64

Electrical Circuits and


DC Machines

Pretest and Post-test Items

Three resistors with resistance 10, 20 and 50 ohms are connected in parallel across a 120-V source.
Find the power dissipated by the 10-ohm resistor.
a. 288 W
c. 180 W
b. 720 W
d. 1 440 W
The power factor in a resistive circuit is _____.
a. leading
c. lagging
b. None of these
d. unity
The load at 0.866 lagging power factor takes a power per phase of 3 000 W. Determine the total
reactive power drawn.
a. 3 289 VARs
c. 1 732 VARs
b. 8 232 VARs
d. 1 021 VARs
How many parallel paths are there in an eight-pole triplex-wave armature?
a. 2
c. 6
b. 4
d. 8
A three-phase induction runs at 290-rpm on full-load. If the line frequency is 50-Hz, calculate the
number of poles and its slip.
a. 10, 6.33%
c. 30, 4.33%
b. 20, 3.33%
d. 40, 7.33%
A heater draws 3 amperes at 12 V dc. How many joules does it consume in 10 minutes?
a. 10 800 J
c. 720 J
b. 21 600 J
d. 12 000 J
This is the part of a dc generator that converts the ac generated emf to dc.
a. field
c. commutator
b. rectifier
d. rotor
If a 12-V car battery has a 0.1 ohm resistance, what terminal voltage causes a 4-amp current to flow
into the positive terminal?
a. 14.2 V
c. 12.4 V
b. 10.2 V
d. 13.6 V
This refers to the rotating field produced by the coils of three-phase inductor motor.
a. actual speed
c. synchronous speed
b. field speed
d. rotor speed
If the waveform of the voltage in a given equation v = 200 sin (377t + 60) volts is shifted to the
right by 60, its phase angle is _____.
a. 0
c. 240
b. 180
d. 120

Electrical Circuits and DC Machines

11
12
13
14
15
16
17
18
19
20

65

Which dc generator is considered to have a constant voltage?


a. induction
c. shunt
b. series
d. compound
Which of the following is not part of magnetic circuit in a dc machine?
a. yoke
c. air gap
b. field coil
d. armature
What is j2 + j + 1 + j3 + j4?
a. j 2
b. j 4

c. j 3
d. 1

A resistor R consumes 100 watts if connected across a 200-volt source. What power will it consume
if the input voltage is doubled?
a. 100 W
c. 400 W
b. 200 W
d. 75 W
A 120-volt, 60-watt lamp is to be connected to a 220-volt, 50-Hz source. Determine the value of a
pure inductor required in order for the lamp to operate on a correct voltage.
a. 2.97 H
c. 5.67 H
b. 1.17 H
d. 4.11 H
Which of the following is not a dc machine?
a. shunt
b. compound

c. series
d. induction

The following are the methods in starting large inductor motor except one.
a. compensator
c. wye-delta
b. phase shifting
d. direct on line
At constant speed, the generator emf E of a dc machine is proportional to _____.
a. frequency
c. poles
b. flux
d. armature current
Which dc motors is not usually started without load?
a. shunt
c. series
b. compound
d. induction
Two resistors Ra = 1.95 ohm and Rb = 0.05 ohm are connected in parallel and take a total of 50 A.
What is the current through each resistor?
a. 2 A, 48 A
c. 1.75 A, 48.25 A
b. 2.75 A, 47.25 A
d. 1.25 A, 48.75 A

66

Electrical Circuits and DC Machines


66

Answers

(d) Hint: (Electric) Power in the Relationship of Voltage and Resistance


V2
P=
R
Eq.E1 is a power dissipation of the resistance in the electrical circuit.

Eq.E1

Given: 10 , 20 , 50 resistors in a parallel connection; 120-V source


Required: the power dissipation of 10
The 20 and 50 resistors are useless. Only determine the power dissipated by the 10 resistor
connected to the voltage source.
P = 1202/10 = 1 440 W

2
3

(d) The power factor in a resistive circuit is unity.


In alternating current (ac) circuit, the (pure) resistor is not a reactive element at which the timevarying voltage and time-varying current are in-phase. If in-phase, the angle is zero. Calculating the
power factor (Eq.E5), that is 1 or unity.
(c) Hint: Power Triangle and Power Factor
V2
P = I 2R =
R
V2
Q = I 2X =
X

S = VI = P2 + Q2
P
pf = = cos
S

Eq.E2
Eq.E3
Eq.E4
Eq.E5

where P = real power (in watt, W)


I = ac root-mean-square (rms) current (in ampere, A)
V = ac rms voltage (in volt, V)
R = resistance (in ohm, )
Q = reactive power (in voltage-amp reactance, VAR)
X = load reactance (in )
S = apparent power (in voltage-amp, VA)
pf = power factor (unitless)
For ac sinusoidal voltages and currents, the real power dissipated in an ac circuit is dissipated in the
resistance only, the resistor (Eq.E2).
There is no real power dissipation in a reactive element such as inductor or capacitor. So the power
in a reactive element is called a reactive power (Eq.E3).
When an ac load has an ac rms voltage V across it and an ac rms current I, the apparent power is
the product of the voltage and current (Eq.E4). It must be no resistance or reactance.
A power triangle (Fig.E1) can be determined where P, Q and S are labeled. Pythagorean Theorem
formulates the three powers together. By applying trigonometry, cos is defined as the power factor

Electrical Circuits and DC Machines

67

and a ratio of the real power divided by the apparent power (Eq.E5).

P
Fig.E1
Given: pf = 0.866 (lagging); P = 3 000 W
Required: Q

The word lagging comes from an inductor but do not mind it because determining the value of
the inductor is not part of the solution. Solve the angle of the power factor.
= cos1 0.866 = 30
Use the power triangle and trigonometry to get Q.
tan = Q/P
Q = P tan = 3 000 tan 30 = 1 732 VARs

(c) Hint: Parallel Path of a Wave Armature


The parallel path a describes the way the machines armature conductors which are connected to
each other and to the number of poles p. The two basic ways of connecting these conductors are
called lap and wave armature winding. It becomes complicated because, in turn, can be connected
in selected n-winding. Table E1 is the construction of the lap and wave armature winding. Table E2
is listed the different terms of the n-winding according to its multiplicity m.

Parallel Path
a

Lap
mp

n-winding
Simplex
Duplex
Triplex
Quadruplex

Wave
2m

Table E1

m
1
2
3
4

Table E2

For triplex-wave armature, the parallel path has


2m = 2(3) = 6

(b) Hint: Synchronous Speed and Slip of a Three-Phase Inductor Motor


120f
Ns =
p

and

Ns Nr
S=
Ns

Eqs.E6 & E7

where Ns = synchronous speed (in revolutions per minute, rpm), f = frequency (in hertz, Hz), p =
number of poles per phase winding, S = normalized slip, Nr = actual rotational speed (in rpm). Eq.E6
is a synchronous speed of the three-phase inductor motor based on the supply frequency and the
number of poles in the motor winding. Actual rpm for an inductor motor will be less than this
calculated Ns by an amount known as slip, that increases with the torque produce; the slip is expressed
in percentage (Eq.E7).

68

Electrical Circuits and DC Machines


Given: f = 50 Hz; Nr = 290 rpm; Ns = 300 rpm
Ns has to be somewhere near the value of Nr; so round off. Make sure that Ns > Nr.
Required: p and S
Solving the number of poles and the slip yields:
120(50)
p = = 20
300
and
300 290
S = = 0.0333 = 33.3%
300

6
7
8

(b) Hint: Power in Relationship of Voltage and Current, and in Relationship of Work and Time
W
P = VI =
t
Given: Vdc = 12 V; I = 3 A; t = 10 min or 600 sec
Required: energy (or work) W of a heater
W = 12(3)(600) = 21 600 J

Eq.E8

(c) The commutator is a part of dc generator that converts the ac generated emf to dc.
(c) Hint: Kirchhoffs Voltage Law (KVL) and Ohms law
V = 0

and

Eqs.E9 & E10

V = IR

The algebraic sum of the potential differences in any loop, including those associated with voltage
sources (or electromotive forces E, emfs) and those of resistive elements, must be zero (Eq.E9).
The applied voltage to a circuit is equal to the product of the current and the resistance of the circuit
(Eq.E10).
Given: Fig.E2 shows the problem.
Required: VT (terminal voltage)
I=4A

E = 12 V

VT
r = 0.1
Fig.E2
Actual sources or emfs like batteries have an internal resistance r. In this problem, there are two
sources: the terminal voltage VT and the battery. This is a basic example of charging a battery. Here,
VT which is a voltage supply must be greater than the voltage of a battery. The polarities of the sources
and elements are important in solving a loop.
There is one loop in this circuit. Applying Kirchoffs voltage law have:
VT E Ir = 0
VT = E + Ir = 12 + 4(0.1) = 12.4 V

Electrical Circuits and DC Machines

69

In the rule of conventional flow, the first source starts at the positive polarity as the current
travels at the positive direction of it. After that, the battery and the voltage of a resistor (by using
Ohms law) go to the negative polarity until it drops to zero. To determine what sign of the polarity
will be, always check out the last terminal except the first source which is the first terminal.

9
10

(c) The synchronous speed refers to the rotating field produced by the coils of three-phase inductor
motor.
(a) Hint: Shifting a Function by a Phase Angle
If the waveform (or function) is shifted to the right, the phase angle is negative; otherwise if it is
shifted to the left, that is positive.
Given: v(t) = 200 sin (377t + 60)
Required: the waveform when shifted to the right by 60
Here, the phase angle is 60. Simply subtract the two phase angles to get the answer.
v(t) = 200 sin (377t + 60 60) = 200 sin 377t
Therefore, the phase angle is zero.

11
12
13

(b) Series is a kind of dc generator that considers to have a constant voltage.


(b) The field coil is not part of a magnetic circuit in a dc machine.
(d) Hint: j-Operator
The j-operator comes from the imaginary number. For electrical engineers, it represents j which

is equal to 1 and is also the angle of 90. The j-operator is used when the multiplication of the
imaginary numbers are successive. Fig.E3 shows it in the phasor diagram as the direction rotates
counterclockwise.
90
+j
j2 = 1
180

+1

j3 = j
270
Fig.E3
As it goes successive, it can be expressed in power. Every power of the imaginary number has the
value of: j1 = j; j2 = 1; j3 = j; j4 = 1. In each successive rotation (or co-terminal angles), the values
remain the same.
Given: j2 + j + 1 + j3 + j4
1 + j + 1 + (j) + 1 = 1

70

Electrical Circuits and DC Machines


If the power is much larger, for example j23, just divide 23 by 4 since there are four quadrants, and
the remainder will be the smallest power. Therefore, j23 = j3 = j. Another example j157, which is not
enough to divide it by 4, is required to select a number less than and near 157 that can be divisible
by 4; then it is 156. Divide 157 by 156 and the remainder is 1. Therefore, j157 = j1 = j.

14

(c) Hint: (Electric) Power (See Eq.E1)


Given: First circuit: PR1 = 100 W; V1 = 200 V

Second circuit: V2 = 400 V
Required: power P of the resistor PR2 in the second circuit
200 V

PR2 = ?
R

100 W
R

400 V

Fig.E4
The two resistors are the same resistance (Fig.E4). Using Eq.E1 in terms of R equates:
V12 V22
R=
=
PR1 PR2
Then,
PR2 =

15

440

200

100 = 400 W

(b) Hint: Phasor Diagram and Inductive Reactance


XL = L = 2fL

and

VL = IXL

Eqs.E11 & E12

where VL = amplitude of voltage across an inductor, XL = inductive reactance, = angular frequency,


L = self-inductance. These formulas are only used in an ac circuit.
Given: Fig.E5a shows the problem.
Required: L
VR = 120 V
60 W
VL
Vac
L
Vac = 220 V
f = 50 Hz
(a)
Fig.E5

VL

VR

(b)

In dc circuit in a loop, KVL is used. In ac circuit in a loop, observe first if the values of the source
and elements have a given complex number or phasor; otherwise do not use KVL. Fig.E5a cannot
be able to apply KVL, so draw a phasor diagram. Since both elements travel with the same phasor of
current, identify where the phasors of voltage will predict. Knowing that in the resistor, the phasors
of voltage and current are in-phase while in the inductor, the phasor of voltage leads the phasor of

Electrical Circuits and DC Machines

71

current by 90. The sum of the phasors of voltage is the phasor of the ac source. Fig.E5b is provided.
Fig.E5b can be drawn into a triangle. Using Pythagorean Theorem obtains:
Vac2 = VR2 + VL2
Then,

2202 1202 184.39 V

VL =

The lamp acts as a resistive element. Getting the current of the circuit, based on the power and voltage
of the lamp, obtains:
I = PR/VR = 60/120 = 0.5 A
Finally for the value of pure inductor L,

16
17
18
19
20

184.39
L = = 1.17 H
2(50)(0.5)

(d) Induction is not a dc machine.


(b) The phase shifting is not operated initially such a large inductor motor.
(b) At constant speed, the generator emf of a dc machine is proportional to flux.
(a) Shunt is not usually started without load in dc motors.
(d) Hint: Current Division
Req
Ix = IT
Rx + Req

Eq.E13

where Ix = current concerned flowing through resistor Rx, IT = total current of the circuit, Req =
equivalent resistance of the parallel circuit expect Rx. Eq.E13 is used only in parallel connection.
Given: Fig.E6 shows the problem.
Required: current through each resistor
IT = 50 A
Ia
Ra = 1.95

Ib
Rb = 0.05
Fig.E6

Solving the two currents yields


50(0.05)
Ia = = 1.25 A
1.95 + 0.05

and

50(1.95)
Ib = = 48.75 A
1.95 + 0.05

72

F
1
2
3
4
5
6
7
8

9
10

Analog Electronics
72

Analog Electronics

Pretest Items

A MOSFET differs from a JFET mainly because _____.


a. of the power rating
c. the MOSFET has two gates
b. the JFET has a pn junction
d. MOSFETs do not have a physical channel
Once the BJT is in saturation, a further increase in the base current will _____.
a. cause the collector current to increase
c. cause the collector current to decrease
b. not affect the collector current
d. turn the transistor off
The high frequency response of an amplifier is determined in part by _____.
a. the gain-bandwidth product
c. the bypass capacitor
b. the internal transistor capacitances
d. the roll-off
If one of the diodes in a bridge full-wave rectifier opens, the output is _____.
a. 0 V
c. a half-wave rectified voltage
b. 1/4 amplitude of the input voltage
d. a 120-Hz voltage
When a differential amplifier is operated in differential-mode, _____.
a. opposite polarity signals are applied to the c. the output are different amplitudes
inputs
d. only one support voltage is used
b. the gain is 1
A 60 V peak full-wave rectified voltage is applied to a capacitor-input filter. If f = 120 Hz, RL = 10
k, C = 10 F, the ripple voltage is_____.
a. 0.6 V
c. 5.0 V
b. 6 mV
d. 2.88 V
In a voltage-divider biased npn transistor, if the lower voltage-divider resistor (the one connected
to ground) opens, _____.
a. the transistor is not affected
c. the transistor may be driven to saturation
b. the transistor may be driven to cutoff
d. the supply voltage is too high
For an op-amp integrator circuit, the rate of change of the output voltage in response to a step input
is set by _____.
a. the RC time constant
c. the current through the capacitor
b. the amplitude of the step input
d. All of these
If the load resistance of a capacitor-filtered full-wave rectifier is reduced, the ripple voltage _____.
a. increases
c. is not affected
b. decreases
d. has a different frequency
A common-emitter amplifier is driving a 10 k load. If RC = 2.2 k and re = 10 , the voltage gain
is approximately _____.

Analog Electronics
a. 220
b. 10

11
12
13
14
15
16
17
18
19
20

73
c. 1 000
d. 180

The output voltage of a zener regulator is 8.23 V at no-load and 7.98 V at full-load. Determine the
regulation expressed as a percentage _____.
a. 3.132%
c. 3.037%
b. 25%
d. 96.962%
A certain inverting amplifier has a closed loop gain of 25. The op-amp has an open loop gain of
100 000. If another op-amp with an open loop gain of 200 000 is substituted in the configuration,
the closed loop gain _____.
a. doubles
c. drops to 12.5
b. remains at 25
d. increases slightly
When you apply a triangular waveform to the input of an op-amp differentiator, the output is _____.
a. a dc level
c. an inverted triangular waveform
b. a square waveform
d. first harmonic of the triangular waveform
When a low-pass and a high-pass filter are cascaded to get a band-pass, the critical frequency of the
pass-filter must be _____.
a. equal to the critical frequency of the highc. greater than the critical frequency of the
high-pass filter
pass filter
b. less than the critical frequency of the high- d. doubled of the critical frequency of the
high-pass filter
pass filter
If the emitter current is 50 times larger than the base current, the alpha is _____.
a. 50
c. 1
b. 0.98
d. 0.96
The operation of a relaxation oscillator is based on _____.
a. the charging and discharging of a capacitor c. a highly selective resonant circuit
b. a very stable supply voltage
d. low power consumption
An oscillator differs from an amplifier because _____.
a. it has more gain
c. it requires no dc signal
b. it requires no input signal
d. it always has the same output
If the base-emitter junction is open, the collector voltage is _____.
c. 0 V
a. VCC
b. floating
d. 0.2 V
The average value of a half-wave rectified voltage with a peak value of 200 V is _____.
a. 63.7 V
c. 141 V
b. 127.3 V
d. 0 V
The positive lead of an ohmmeter is connected to the cathode of a diode, and the negative lead is
connected to the anode. The diode is _____.

74

Analog Electronics
a. reverse-biased
b. open

21
22
23
24
25
26
27
28
29
30

c. forward-biased
d. faulty

Post-test Items

Related from Item 7: In a voltage-divider biased npn transistor, if the upper voltage-divider resistor
(the one connected to VCC) opens, _____.
a. the transistor burns out
c. the transistor goes into saturation
b. the supply voltage is too high
d. the transistor goes into cutoff
Related from Item 19: the average value of a full-wave rectified voltage with a peak value of 200 V
is _____.
a. 63.7 V
c. 0 V
b. 141 V
d. 127.3 V
Related from Item 1: A D-MOSFET differs from an E-MOSFET mainly _____.
a. the absence of channel in E-MOSFET
c. of the power rating
b. the D-MOSFET has two gates
d. the D-MOSFET has a pn junction
Line regulation is determined by _____.
a. changes in output and input voltage
b. load current

c. changes in load current and input voltage


d. zener current and load current

For operation as an amplifier, the base of an npn transistor must be _____.


a. positive with respect to the collector
c. negative with respect to the emitter
b. positive with respect to the emitter
d. zero
Related from Item 20: The anode of the diode is connected to the positive source and the cathode
to the negative source. The diode is _____.
a. open
c. forward-biased
b. reverse-biased
d. faulty
Related from Item 9: If the capacitor of a capacitor-filtered full-wave rectifier is increased, the ripple
voltage _____.
a. has a different frequency
c. is not affected
b. increases
d. decreases
Which of the following characteristics does not necessarily apply to an op-amp?
a. high gain
c. high input impedance
b. low power
d. low output impedance
Related from Item 10: A common-emitter amplifier is driving a 15 k load. If RC = 3.3 k and
re = 8 , the voltage gain is approximately _____.
a. 338
c. 412.5
b. 500
d. 188
Related from Item 4: If two of the diodes in a bridge full-wave rectifier opens, the output _____.
a. can be a half-wave rectified voltage
c. is a 120-Hz voltage
b. is 1/4 amplitude of the input voltage
d. is 0 V

Analog Electronics

31
32
33
34
35
36

75

One condition for oscillation is _____.


a. a phase shift around the feedback loop of 0 c. a gain around the feedback loop of one-third
b. a phase shift around the feedback loop of 180 d. a gain around the feedback loop of less than 1
Related from Item 14: When a low-pass and a high-pass filter area cascaded so that the critical
frequency of the low-pass is lower than the high-pass, it will create what type of filter.
a. band-pass filter
c. second order high-pass filter
b. notch filter
d. second order low-pass filter
Related from Item 13: When you apply a square waveform to the input of an op-amp diffferentiator,
the output is _____.
a. voltage spikes
c. a triangular waveform
b. an inverted triangular waveform
d. a dc level
The feedback element of an op-amp differentiator is a _____.
a. voltage divider
c. capacitor
b. resistor
d. zener diode
Related from Item 3: The low-frequency response of an amplifier is determined in part by _____.
a. the bypass capacitor
c. the gain-bandwidth product
b. the internal transistor capacitances
d. the roll-off
Related from Item 6: A 60 V peak full-wave rectified voltage is applied to a capacitor-input filter.
If f = 60 Hz, RL = 10 k, C = 10 F, the ripple voltage is_____.
a. 5.0 V
c. 10 V
b. 6 mV
d. 0.6 V

76

F
1
2
3
4

Analog Electronics
76

Answers

(b) The MOSFET (metal-oxide semiconductor field-effect transistor) differs from a JFET (junction
field-effect transistor) mainly because the JFET has a pn junction.
(b) Once the BJT (bipolar junction transistor) is in saturation, a further increase in the base current
will not affect the collector current.
It is because to do a forward bias of npn BJT, the current from the base always flows to the emitter.
(b) The high frequency response of an amplifier is determined in part by the internal transistor
capacitances.
Actual transistors have internal capacitances which can cause their behavior to depart from that of
ideal transistor. It is located in each between-terminal junctions of the transistor.
(c) If one of the diodes in a bridge full-wave rectifier opens, the output is a half-wave rectified
voltage.
Bridge full-wave rectifier contains four diodes with the configuration as Fig.F1a. The ac input voltage
vi has a time-varying function such as a sinusoidal wave (Fig.F1b). To get the output voltage vo, just
analyze the circuit on the operation. Observe the current flow between the positive and negative
region of the input together with the forward bias of the ideal diodes and the polarity of the load,
that is at vo (Figs.F1c and F1d). Both regions result equally positive output voltages (Fig.F1e).
Configuration

D1

vi

D2
R

vi

D3

D4

(a)

(b)
Operation

D2

vo
D1

vo

vo

vi

vi

D3
(c)

D4
(d)

(e)

Fig.F1

Analog Electronics

77

If one of the diodes is removed, either positive or negative region of the input will not conduct and
that voltage distorts at zero (Fig.F2). These are called half-wave rectified voltage which is the same
output with the half-wave rectifier.
vo

vo

Fig.F2

5
6

(a) When a differential amplifier is operated in differential-mode, opposite polarity signals are
applied to the inputs.
Hint: Formula of a Ripple Voltage
1
Vr(pp) = Vm
fRLC

Eq.F1

where Vr(pp) = ripple peak-to-peak voltage, Vm = maximum peak voltage, RL = resistance of the load,
C = capacitance. Eq.F1 is used for the circuit having a capacitor-input filter.
Given: Vm = 60 V; f = 120 Hz; RL = 10 ; C = 10 F
Required: Vr(pp)
60
Vr(pp) =
=5V
120(10 103)(10 106)

7
8
9
10

(c) In a voltage-divider biased npn transistor, if the lower voltage-divider resistor (the one connected
to ground) opens, the transistor may be driven to saturation.
There are two resistors, denoted as R1 and R2, connected in parallel network in the voltage-divider
configuration. Focusing at the base current, it flows first through R1, and then it divides to the R2
and to the base terminal of the transistor. If R2 opens, the base current instantly flows through R1
to the base terminal as the base current much increases.
(a) For an op-amp integrator circuit, the rate of change of the output voltage in response to a step
input is set by the RC time constant.
(a) If the load resistance of a capacitor-filtered full-wave rectifier is reduced, the ripple voltage
increases.
You can test Eq.F1 to determine Vr(pp) with the assigned values of the quantities.
(d) Hint: Formula of a Voltage Gain in CE (Common-Emitter) Amplifier
RC||ro
AV =
re

Eq.F2

where AV = voltage gain, RC = resistor connected to the collector terminal of the transistor, ro =
transistor output resistance, re = transistor emitter resistance. Eq.F2 is derived based on the transistor
modeling, construction of the ac equivalent circuit of the amplifier, and dc biasing.

78

Analog Electronics
Given: RC = 2.2 k; ro = 10 k; re = 10
Required: AV
The RC and ro are connected in parallel.
(10 103)(2.2 103)
RC ||ro =
1803
10 103 + 10 106
Then,
AV = 1803/10 180

11

(a) Hint: Voltage Regulation


VNL VFL
VR =
VFL

Eq.F3

where VNL = no-load voltage, VFL = full-load voltage. Eq.F3 is a factor (in a power supply) for which
the amount the dc output voltage changes over a range of circuit operation.
Given: VNL = 8.23 V; VFL = 7.98 V
Require: VR
8.23 7.98
VR = = 0.03132 = 3.132%
7.98

12

(b) Hint: Closed Loop Gain with an Open Loop Gain of an Inverting Amplifier
Rf
Aclosed =
Ri + R f
Ri +
Aopen

Eq.F4

where Rf = feedback resistor, Ri = resistor connected to input voltage. Eq.F4 is derived based on the
same input voltage of the closed loop and open loop of an inverting amplifier. The difference of the
two loops is that the closed loop has a feedback. Because the open loop is so easily saturated, the
gain is too high. So, Eq.F4 becomes:
R
Aclosed = f
Ri
Concluding that the closed loop gain will not affect with the open loop gain. Open loop op-amp
circuits are not very useful. Based on the given, the answer is the closed loop gain remains at 25.

13
14
15

(b) When you apply a triangular waveform to the input of an op-amp differentiator, the output is a
square waveform.
This can be also proven mathematically by applying Fourier series and derivative.
(c) When a low-pass and a high-pass filter are cascaded to get a band-pass, the critical frequency of
the pass-filter must be greater than the critical frequency of the high-pass filter.
(b) Hint: Alpha and Beta of the npn BJT Transistor, and Single Node
= IC/IE
= IC/IB
IE = IC + IB

Eq.F5
Eq.F6
Eq.F7

Analog Electronics

79

where IC = collector current, IE = emitter current, IB = base current. In the dc mode under the condition
of the common-base configuration, the levels of IE due to the majority carriers are related by
a quantity called alpha (Eq.F5). In the dc mode under the condition of the common-emitter
configuration, the levels of IC and IB are related by a quantity called beta (Eq.F6). All configurations
are applied with Kirchhoffs current law to the transistor as if it were a single node. (Eq.F7).
Given: IE = 50 A; IB = 1 A
Required:
Letting IC = IE, the Eq.F6 becomes:
Then,

16
17
18
19

IE = IE + IB
50 1
= = 0.98
50

(a) The operation of a relaxation oscillator is based on the charging and discharging of a capacitor.
(b) An oscillator differs from an amplifier because it requires no input signal.
The oscillator is supplied with dc voltage and can feed its own input.
(a) If the base-emitter junction is open, the collector voltage is VCC.
This situation makes the transistor not to conduct at all including the circuit itself because the emitter
is connected to the ground.
(a) Hint: Formula of an Average Value of a Half-wave Voltage
Vave = Vm/

Eq.F8

where Vave = average voltage, Vm = maximum peak voltage. The term average is typically a dc voltage
level. Eq.F8 is derived using an integral.
Given: Vm = 200 V
Required: Vave
Vave = 200/ = 63.7 V

20
21
22

(c) The diode is forward-biased when the positive lead of an ohmmeter is connected to the cathode,
and the negative is connected to the anode.
(d) In a voltage-divider biased npn transistor, if the upper voltage-divider resistor (the one connected
to VCC) opens, the transistor goes into cutoff.
The collector resistor RC, not R1, is now disconnected. Supposedly IC must be greater than IB; however
the problem makes IC not to flow off and IB still has a very low current. The transistor is not enough
to saturate and is considered to be part in a cutoff region.
(d) Hint: Formula of an Average Value of a Full-wave Voltage
Vave = 2Vm/
Given: Vm = 200 V

Eq.F9

80

Analog Electronics
Required: Vave
2(200)
Vave = = 127.3 V

23
24
25
26
27
28
29
30
31
32
33
34
35

(a) A D-MOSFET (depletion-type MOSFET) differs from an E-MOSFET (enhancement-type


MOSFET) mainly the absence of channel in E-MOSFET.
(a) Line regulation is determined by changes in output and input voltage.
(b) For operation as an amplifier, the base of an npn transistor must be positive with respect to the
emitter.
This condition is relevant concept in Item 2.
(b) The diode is reverse-biased when the anode of the diode is connected to the positive source and
the cathode to the negative source.
(d) If the capacitor of the capacitor-filtered full-wave rectifier is increased, the ripple voltage
decreases.
(b) Low power consumption is not necessary to apply to an op-amp.
It is because the output will produce a poor qualify of amplification or buffering.
(a) Use Eq.F2.
(a) & (d) If two of the diodes in a bridge full-wave rectifier opens, the output can be a half-wave
rectified voltage, or it is 0 V.
Just analyze in Fig.F1. If D2 and D3, or D1 and D4 are removed, the output is a half-wave rectified
voltage. If D2 and D3, or D2 and D4 are removed, the circuit does not conduct at all and the voltage
is zero.
(b) One condition for oscillation is a phase shift around the feedback loop of 180.
(b) That is a notch filter when a low-pass and a high-pass filter are cascaded so that the critical
frequency of the low-pass is lower than the high-pass.
(a) When you apply a square waveform to the input of an op-amp differentiator, the output is voltage
spikes.
(b) The feedback element of an op-amp differentiator is a resistor.
(a) The low-frequency response of an amplifier is determined in part by the bypass capacitor.

Analog Electronics

81

Very less or less internal transistor capacitances activate in a low frequency. Instead the bypass (or
coupling) capacitors enact on the operation. They are usually active filters which help to get rid of
ac signal or ac noise by passing freely the dc signal through the load.

36

(c) Use Eq.F1.

82

1
2
3
4
5
6
7
8
9
10
11

Digital Electronics
82

Digital Electronics

Pretest Items

What is the major contribution in the invention of an IC?


a. size
c. complication
b. power consumption
d. speed
What is a group of circuits that provide timing and signals to all operations in the computer?
a. output unit
c. control unit
b. memory unit
d. input unit
Which of the following is not a type of flip-flop?
a. RS
c. D
b. latch
d. register
In a clocked sequential logic circuit, what memory element is being used?
a. flip-flop
c. gates
b. read-only memory
d. static-RAM
A gate that produces an output only if pulses are sent through both wires the same time.
a. NAND gate
c. NOR gate
b. AND gate
d. OR gate
A solid-state logical device which only gives a 1 output if all inputs are 0 is called a _____.
a. NOT gate
c. NAND gate
b. NOR gate
d. OR gate
A digital word consisting of only four bits is called a _____.
a. dibit
c. pixel
b. quad
d. nibble
Speed of a logic circuit is normally expressed as _____.
a. logic levels
c. propagation delay
b. speed immunity
d. power consumption
When used with an IC, what does the term quad indicate?
a. 2 circuits
c. 6 circuits
b. 4 circuits
d. 8 circuits
The devices that provide the means for a computer to communicate with the user or other computers
are referred to as _____.
a. CPU
c. I/O
b. ALU
d. None of the above
The time required for a gate or inverter to change its state is called _____.
a. rise time
c. propagation time
b. delay time
d. charging time

Digital Electronics

12
13
14
15
16
17
18
19

83

Odd parity of word can be conveniently tested by _____.


a. OR gate
c. NOR gate
b. AND gate
d. XOR gate
Which of the following will give the sum of full adders as output?
a. 3-point majority circuit
c. 3-bit comparator
b. 3-bit parity circuit
d. 3-bit counter
Which of the following statements is wrong?
a. Propagation delay is the time required for a c. Fan-in of a gate is always equal to fan-out of
the same gate.
gate to change its state.
b. Noise immunity is the amount of noise
d. Operating speed is the maximum frequency
which can be applied to the input of the gate
at which digital data can be applied to a
without causing the gate to change state.
gate.
A one-to-four demultiplexer is to be implemented using a memory. How many bits must each word
have?
a. 1 bit
c. 4 bits
b. 2 bits
d. 8 bits
Which table shows the logical state of a digital circuit output for every possible combination of logical
states in the inputs?
a. function table
c. routing table
b. truth table
d. ASCII table
A demultiplexer is used to _____.
a. route the data from single input to one of
the many outputs
b. perform serial to parallel conversion

c. Both a and b
d. select data from several inputs and route it
to single output

How many full adders are required to construct an m-bit parallel adder?
a. m/2
c. m
b. m 1
d. m + 1
To make the following circuit in Fig.G1, a tautology ? marked box should be _____.
x

x+y

f = (x + x ) + (y + y )

y
Fig.G1
a. OR gate
b. AND gate

20

If A B = C, then _____.

c. NAND gate
d. XOR gate

84

Digital Electronics
a. A C = B
b. B C = A

21
22
23
24
25
26

Post-test Items

Which of the following set of gates are best suited for parity checking and parity generation?
a. NAND gates
c. XNOR or XOR gates
b. AND, OR, NOT gates
d. NOR gates
The digital multiplexer is basically a combinational logic circuit to perform the operation.
a. OR-AND
c. AND-AND gates
b. OR-OR operation
d. AND-OR gates
If a logic gates has four inputs, then the total number of possible input combinations is _____.
a. 32
c. 8
b. 4
d. 16
A combinational logic circuit which generates a particular binary word or number is _____.
a. decoder
c. multiplexer
b. demultiplexer
d. encoder
Parallel adders are _____.
a. sequential logic circuits
b. combinational logic circuits

c. Both a and b
d. None of these

The box in Fig.G2 consists of a minimum complexity circuit that uses only AND, OR, and NOT
gates. The function f(x, y, z) = 1 when x, y are different and 0 otherwise. In addition, the 3 inputs
x, y, z are never all the same value. Which of the following equations lead to the correct design for
the minimum complexity circuit?

27
28
29

c. A B C = 0
d. Both a and b

x
y
z

f(x, y, z)
Fig.G2

a. xyz + xyz
b. x + yz

c. xy + xy
d. xy + yz + z

OR gate can be imagined as _____.


a. switches connected in series
b. MOS transistors connected in series

c. switches connected in parallel


d. None of the above

What is the largest number of data inputs which data selector with two control inputs can have?
a. 16
c. 8
b. 2
d. 4
Which of the following logic expression is incorrect?
a. 1 1 1 = 1
c. 1 0 = 1
b. 1 1 = 0
d. 1 1 0 = 1

Digital Electronics

30
31
32
33
34
35
36
37
38
39
40

85

Adder _____.
a. is called so because a full adder involves two c. needs two inputs and generates two outputs
half-adders
d. All of these
b. adds 2 bits
The output of NOR gate is _____.
a. low if one of its inputs is low
b. high if all of its inputs are high

c. low if all of its inputs are low


d. high if all of its inputs are low

Which of the following circuit can be used as parallel to serial converter?


a. decoder
c. digital counter
b. multiplexer
d. demultiplexer
The function of a multiplexer is _____.
a. to transmit data on N lines
b. to decode information

c. to perform serial to parallel conversion


d. to select 1 out of N input data sources and to
transmit it to single channel

Which combination of gates does not allow the implementation of an arbitrary Boolean function?
a. NAND gates only
c. OR gates and XOR gates only
b. OR gates and NOT gates only
d. OR gates and AND gates only
AND circuit _____.
a. is a memory circuit
b. is a negative OR gate

c. gives an output when all input signals are


present simultaneously
d. is a linear circuit

In which of the following adder circuits, the carry look ripple delay is eliminated?
a. half adder
c. parallel adder
b. carry look ahead adder
d. full adder
What is the minimum number of 2-input NAND gates required to implement the function F = (x +
y)(z + w)?
a. 5
c. 6
b. 4
d. 3
How many truth tables can be made from one function table?
a. any numbers
c. 2
b. 3
d. 1
A combinational circuit is one in which the output depends on the _____.
a. input combination at the time
c. present output and the previous output
b. input combination and the previous output d. input combination at the time and the
previous input combination
A comparison between serial and parallel adder reveals that serial order _____.
a. is faster
c. is more complicated
b. operates at the same speed as parallel
d. is slower

86

1
2
3
4
5

Digital Electronics
86

Answers

(a) The size of an IC (integrated circuit) is the major contribution in the invention.
The materials matter the cost to form an IC. The manufacturer minimizes the size to produce more
ICs where the patterns inside are measured in nanometers that cannot be seen on the naked eye.
The thickness of a pattern on each layer is even lesser than the diameter of a one red blood cell.
(c) The control unit is a group of circuits that provide timing and signals to all operations in the
computer.
(d) Register is a group of cascaded flip-flop used to store related bits of information.
(a) Flip-flop is used as a memory element in a clocked sequential logic circuit.
(b) That is AND gate.
Common logic gates are NOT, AND, OR, NAND, and NOR gates as shown in Fig.G3 respectively.
AND and OR gates are inverted form of NAND and NOR gates. In digital, there are two binary
numbers: 0 and 1. Basically these logic gates have been familiarized their outputs having two
input functions provided in Table G1 which is called a truth table. Two input functions have possibly
four input combinations except NOT gate.

Fig.G3
NOT (or inverter) gate has only one input; then the output is opposite from it.
Input
A

0
0
1
1

0
1
0
1

AND
AB = Y
0
0
0
1

Output Y
OR
NAND

A+B=Y
AB = Y
0
1
1
1
1
1
1
0

NOR

A+B=Y
1
0
0
0

Table G1
In the problem, the condition states that only one logic gate produces an output if the pulses are sent
through both inputs the same time. A pulse refers to an input binary 1. Doing a short connection
with one input of each gate, Fig.G4a shows the correct output which is AND gate.
OR gate can generate 1, however its output produces a 1 from other input combinations which
does not follow the condition.

(b) That is NOR gate.

Digital Electronics

87

The condition states that only one logic gate gives an output of 1 if all inputs are 0; that is NOR
gate as shown in Fig.Gd. NAND gate can generate 1, however its output produces a 1 from other
input combinations which does not follow the condition.
pulse, 1
(a)

(b)

(c)

(d)

Fig.G4

7
8
9

(d) A digital word consisting of only four bits is called a nibble.


(c) The speed of a logic circuit is normally expressed as propagation delay.
(b) The term quad indicates that there are 4 circuits in an IC.
One of the example here is the Quad 2-input AND gate with a code name of 7408. Fig.G5 shows
the structure and the connection diagram of that IC where four circuits of AND gate are indicated.

pin

10
11
12

Fig.G5

(c) I/O (input/output) refers to the devices that provide the means for a computer to communicate
with the user or other comunicate with the user or other computers.
(c) The time required for a gate or inverter to change its state is called propagation time.
(d) That is XOR gate.
The truth table of a 2-input XOR logic gate is listed in Table G2.
Input
A

0
0
1
1

0
1
0
1

Output Y
XOR

B= AB=Y
AB + AB
0
1
1
0
Table G2

88

Digital Electronics
Parity means that the two input conditions are equal; otherwise, if one condition is opposed, it is
an odd parity. Only XOR gate gives an output 1 from the input combinations of 01 and 10 which
is an input odd parity.

13
14
15
16
17
18
19

(d) Among three choices, 3-bit counter gives the sum of full adders as output.
Counters are used for counting a sequence of binaries. Full adders are cascaded to be a counter in
order to implement an addition operation by how many bits can produce. The disadvantage is when
the cascade increases, the process overloads to obtain the output.
(c) The statement in Letter c is incorrect.
Fan-in and fan-out are terms that define the maximum number of digital inputs and outputs that a
single logic gate can accept. Fan-in of a gate is not all equal to its fan-out. For example, multiplexer
can have more inputs (or fan-ins) with one or least outputs (or fan-out). Some can be equal such as
NOT gate which has one input and one output.
(a) Each word has a one bit in a one-to-four demultiplexer as a memory.
The word can be referred as an output containing a number of bits. Because there is only one input
of that demultiplexer, it also produces every single bit.
(b) The truth table shows the logical state of a digital circuit output for every possible combination
of logical states in the inputs.
(c) A demultiplexer is used to route the data from single input to one of the many outputs, and/
or to perform series to parallel conversion.
(b) An m-bit parallel adder must be constructed an m 1 of full adders.
For example, a 4-bit parallel adder comprises of one full adder and three half adders.
(c) Hint: Applying the Laws of Boolean Algebra
Given: Go to Fig.G1.
Require: Find the missing logic gate.
The output f is arranged using commutative law.


f = (x + x ) + (y + y ) = (x + y) + ( x + y )
Notice that (x + y) from the output f is OR gate which is part in the circuit. Next is to find out what

the logic gate will be in the form of ( x + y ).

Since NAND gate defaults as fNAND = xy , De Morgans theorem is applied as:



fNAND = x + y
Therefore, the missing part is NAND gate.

20

(d) If A B = C, then A C = B or B C = A.
The binary operation of XOR gate can be a commutative law. The trials test arbitrarily as follows:
1) Let A = 0 and B = 1; therefore C = 1.
2) So if A = 0 and C = 1, then B = 1.
3) Also if B = 1 and C = 1, then A = 0.

Digital Electronics

21
22
23

89

(c) XNOR or XOR gates are the best suited for parity checking and parity generation.
(d) Digital multiplexer is basically a combinational logic circuit that performs AND-OR operation.
This can be interpretted through a logic equation. For example, having the output equation of AB +
CD, you have to solve first AND operation and then OR operation.
(d) Hint: Input Combinations in Binary
where m = input combinations, n = number of input functions. Eq.G1 is to determine the total number
of possible combinations of binary inputs to a gate.
Given: n = 4

24
25
26

Eq.G1

m = 2n

m = 24 = 16

(a) The decoder is a combinational logic circuit which generates a particular binary word or number.
(b) Parallel adders are combinational logic circuits.
Hint: Karnaugh Map (K-map)
Karnaugh Map is a method to make an output equation of logic gate circuit, which is easier than
Boolean algebra. The advantage of this method is to minimize the number of logic gates.
Given: Go to Fig.G2.
Required: the output equation
The three inputs x, y, and z must have 8 input combinations, which is from 23 = 8. Predict the truth
table (Table G3) according to the statement of the problem. K-map having an 8 input combinations
looks like in Fig.G6a.

A
0
0
0
0
1
1
1
1

Input
B
0
0
1
1
0
0
1
1

C
0
1
0
1
0
1
0
1

Output
f(x, y, z)
0
0
1
1
1
1
0
0

The inputs x and y are


different. The output f(x, y, z) is 1.
The inputs x and y are the same.
The output f(x, y, z) is 0.
The inputs x, y, and z are the
same. The output f(x, y, z) is 0.

Table G3

Fill out the binary outputs in each square as provided in Fig.G6b. Draw a loop with the pair of 1s
and there are two of them (Fig.G6c). Take note that pairing or grouping 1s is symmetric or even.
Anyways, there are different ways to loop a valid pair.

90

Digital Electronics
In the rule of sum-of-products (or SoP) each loop must represent a product; this will be found
out on the preceding steps.

(a)

attribute

(b)

Fig.G6

(c)

The first loop L1 at the left, locates between 01 xy-attribute, and 0 and 1 z-attribute. Reject
the attribute if the loop contradicts unequal binary rows or columns. So the rejected attribute of
L1 is z-attribute which has 0 and 1 unequally. Change the remaining attribute from the binary

numbers into variables. The binary 0 is similar to NOT gate. The product finally is L1 = xy .

Knowing the method, for instance, the second loop is L2 = xy.


Following the SoP, the logic equation is:

f(x, y, z) = L1 + L2 = xy + xy
The symbol prime on the choices is just a negation similar with the symbol bar.

27
28
29
30
31
32
33
34

(c) OR gate can be imagined as switches connected in parallel.


This can be referred to MOS (metal-oxide semiconductor) transistor network of a logic gate.
(d) Using Eq.G1 with n = 2, 22 = 4.
(d) The logic expression 1 1 0 = 1 is incorrect.
You can test it in the truth table of XOR gate (Table G2).
(d) All answers are correct.
(d) The output of NOR gate is high if all of its inputs are low.
(b) The multiplexer can be used as parallel to serial converter.
Multiplexers are also hierarchy blocks that help to reduce complexity of group circuits.
(d) The function of a multiplexer is to select one out of the number of input data sources and to
transmit it to single channel.
(d) OR and AND gates are not allowed the implementation of an arbitrary Boolean function.
The logic gate(s) might be reversible between the input and the output itself using the arbitrary
Boolean function as a tautological condition.
One of the better implementation is NAND gate. Let A = 0 and B = 1 arbitrarily, then the output Y =

AB = 1. Storing our assumed case, we reverse it in any manner, and say: YB = A; then A = 0.
When two logic gates are connected, this is suggested to use MOS transistor network either in parallel

Digital Electronics

91

or series connection in order to test if they are reversible.

35
36
37

(c) AND circuit gives an output when all input signals are present simultaneously.
(c) The parallel adder eliminates the carry look ripple delay.
(b) Hint: De Morgans Theorem


AB = A + B

A+B=AB

AB = AB = A + B

A+B=A+B=AB

Eq.G2

The following logic equations are applied using the De Morgans Theorem (Eq.G2).

Given: F = ( x + y )(z + w)
Required: the number of NAND gates
If the inputs are A and B, NAND gate is AB. To derive the logic equation, no + signs will exist
at the last solution.

F = ( x + y )(z + w)

= xy (z + w)

= ( xy )z + ( xy )w

= ( xy )z + ( xy )w

F = [( xy )z] [( xy )w]

Therefore, there are 4 NAND gates in all: (1) xy , (2) ( xy )z , (3) ( xy )w , and (4) [( xy )z] [( xy )w] .

38
39
40

(c) Using Eq.G1 with n = 1, 21 = 2.


(a) A combination circuit is one in which the output depends on the input combination at the time.
(d) A comparison between serial and parallel adder reveals that serial order is slower.
To find out more about this test, go to source:
http://www.avatto.com/computer-science/test/digital-electronics/mcqsheadlist/6.html

92

1
2
3
4
5
6

7
8
9
10

92

Industrial Electronics

Pretest Items

In a TRIAC full-wave phase control circuit, power is delivered to the load _____.
a. during both half-cycles when the TRIAC is c. for the positive half-cycle only
conducting
d. continuously
b. during the positive half-cycle when the
TRIAC is conducting

Holding current IH in an SCR is the _____.


a. minimum gate current required to keep the
SCR conducting
b. minimum anode current required to keep
the SCR conducting

c. maximum gate current required to keep the


SCR conducting
d. maximum anode current required to keep
the SCR conducting

In order to conduct, an SCR has to be _____.


a. reverse biased, gate to anode
b. reverse biased, gate to cathode

c. forward biased, gate to anode


d. forward biased, gate to cathode

An SCR is a _____.
a. common emitter transistor
b. normal rectifier

c. normal diode
d. thyristor

A UJT is a _____.
a. resistance controlled device
b. voltage controlled device

c. None of the above


d. current controlled device

In a phase control circuit, rms power to the load is decreased by _____.


a. an increase in the conduction angle of the
c. an increase in output voltage
load signal
d. a decrease in the phase retard of the load
signal
b. a decrease in the conduction angle of the
load signal
The minimum emitter voltage needed to cause the UJT to fire is the _____.
a. phase retard
c. intrinsic standoff ratio
b. peak-point emitter voltage or peak voltage
d. gate-turn
An SCR has _____.
a. three layers arranged as npn
b. three layers arranged as pnp

c. four layers arranged as pnpn


d. two layers arranged as pn

In a TRIAC, MT1 is an abbreviation for _____.


a. main terminal 1
c. minor terminal 1
b. the specific manufacturers terminal number d. None of the above
The three terminals of a UJT are _____.

Industrial Electronics
a. A, K, and G
b. E, G, and C

11
12
13
14
15
16
17
18
19
20

93
c. E, B1, and B2
d. E, B, and C

In a power control application, an SCR may be required to act as the _____.


a. controlling device only
c. rectifying device only
b. All of the above
d. controlling as well as the rectifying device
The only junction of an SCR that measures like a normal diode is the _____.
a. anode-cathode junction
c. gate-anode junction
b. cathode-gate junction
d. gate-drain junction
The anode current of an SCR is limited by the _____ (constant voltage source).
a. current source
c. anode resistor
b. power source
d. gate to cathode bias voltage
The three terminals of an SCR _____.
a. MT1, MT2, and G
b. E, B, and C

c. A, B1, and K
d. A, K, and G

A TRIACs conduction can be described as _____.


a. omnidirectional
c. bidirectional
b. bilateral
d. unidirectional
Anode current in an SCR can be determined by the _____.
a. device specification sheet
c. anode resistor value and the anode resistor
b. source voltage and gate resistor values
voltage
d. gate voltage and anode-cathode resistance
Specific SCR holding current can be determined from the value of _____.
a. VAK and the anode resistor
c. the anode resistor voltage (just before SCR
b. the gate resistor
turns-off) divided by the anode resistor value
d. the gate voltage divider resistor
The amount of time an SCR conducts in electrical degrees is described as the _____.
a. phase retard
c. conduction angle
b. conduction phase
d. phase angle
Which of the following statement is true of SCR phase control?
a. The SCR alone determines the power to the c. The load determines the conduction angle.
load.
d. Power can be controlled to a load.
b. As the conduction angle across the load
increases, average voltage decreases.
For an SCR, a small gate voltage can _____.
a. turn off the device
b. destroy the device

c. perform half-wave rectification


d. control a large anode-cathode voltage

94

21
22
23
24
25
26
27
28
29
30

Industrial Electronics
Additional Items (Not Part in the Exam)

The TRIAC turns on when a proper gate trigger voltage is applied. The TRIAC turns off when the
_____.
a. conduction angle is maximum
c. gate trigger is removed
b. gate is reverse triggered from decreasing
d. load signal drops below the voltage needed
signal at the gate
to supply holding current IH to the TRIAC
For an SCR to turn on, the _____. (Assume forward bias voltage is applied.)
a. gate has to be more positive than the
c. anode has to be more positve than the
cathode
cathode
b. gate has to be more positive than the anode d. anode has to be more positive than the gate
Parameters unique to a particular thyristor can be found on the manufacturers _____.
a. schematic
c. specification sheet
b. label
d. price list
For an SCR to turn off, the gate _____.
a. must be more negative than the anode
b. has nothing to do with SCR to turn-off

c. must be more negative than the cathode


d. must be more positive than the anode

Decreasing the conduction angle _____.


a. decreases the gate trigger voltage
c. increases the power across the load
b. decreases the average voltage across the load d. increases the current through the load
An SCR used as a half-wave rectifier conducts _____.
a. only on positive alternations of an ac signal c. only on negative alternations of dc signal
b. on both positive and negative alternations of d. None of the above
an ac signal
A TRIAC operates much like two inversely parallel _____.
a. npn transistors
c. UJTs
b. pnp transistors
d. SCRs
The output signal of a UJT at B1 is a _____.
a. sawtooth
b. pulse

c. sine wave
d. square wave

A UJT has _____.


a. one pn junction
b. one pn and one np junction

c. two pn junctions
d. four pnpn layers

In how many triggering modes can a TRIAC operate?


a. four
c. two
b. three
d. one

H
1
2
3
4
5
6
7
8
9
10
11
12

95

Answers

(a) In a TRIAC (triode for alternating


current) full-wave phase control circuit,
power is delivered to the load during
both half-cycles when the TRIAC is
conducting.

(b) Holding current IH in an SCR (silicon


controlled rectifier) is the minimum
anode current required to keep the SCR
conducting.
(d) In order to conduct, an SCR has to be
forward biased from gate to cathode.
(d) An SCR is a thyristor.
(b) A UJT (unijunction transistor) is a
voltage controlled device.
(b) In a phase control circuit, rms (rootmean-square) power to the load is
decreased by a decrease in the conduction
angle of the load signal.
(b) The minimum emitter voltage needed
to cause the UJT to fire is the peak-point
emitter voltage or peak voltage.
(c) An SCR has four layers arranged as
pnpn.
(a) In a TRIAC, MT1 is an abbreviation for
main terminal 1.
(c) The three terminals of a UJT are E
(emitter), B1 (base 1), and B2 (base 2).
(a), (b), (c) & (d) All choices
(b) The only junction of an SCR that
measures like a normal diode is the
cathode-gate junction.

13
14
15
16
17
18
19
20
21
22
23

(c) The anode current of an SCR is limited


by the anode resistor if the voltage source
is constant.
(d) The three terminals of an SCR are A
(anode), K (cathode), and G (gate).
(c) A TRIACs conduction can be described
as bidirectional.
(c) Anode current in an SCR can be
determined by the anode resistor value
and the anode resistor voltage.
(c) Specific SCR holding current can be
determined from the value of the anode
resistor voltage (just before SCR turnsoff) divided by the anode resistor value.
(c) The amount of time an SCR conducts
in electrical degrees is described as the
conduction angle.
(d) Among the choices, power can be
controlled to a load in the SCR phase
control.
(d) For an SCR, a small gate voltage can
control a large anode-cathode voltage.
(d) The TRIAC turns on when a proper
gate trigger voltage is supplied; it turns off
when the load signal drops below the
voltage needed to supply holding current
to the TRIAC.
(a) For an SCR to turn on, the gate has to
be more positive than the cathode, when
the forward bias voltage is applied.
(c) Parameters unique to a particular
thyristor can be found on the
manufacturers specification sheet.

96

24
25
26
27
28
29
30

Industrial Electronics
(b) For an SCR to turn off, the gate has
nothing to do.
(b) Decreasing the conduction angle
decreases the average voltage across the
load.
(a) An SCR used as a half-wave rectifier
conducts only on positive alternations of
an ac signal.
(d) A TRIAC operates much like two
inversely parallel SCRs.
(b) The output signal of a UJT at B1 is a
pulse.
(a) A UJT has one pn junction.
(a) A TRIAC can operate four triggering
modes.

Computer Networks and


Data Communications: Part 1

97

Abbreviations which are mentioned in the items are defined.



AWG American Wire Gauge

CMD Command Prompt (usage in Windows Operating System)

CRC Cyclic Redundancy Check

CSMA Carrier Sense Multiple Access

DHCP Dynamics Host Configuration Protocol

DNS Domain Name System

FTP File Transfer Protocol

HTTP Hypertext Transfer Protocol

IEEE Institute of Electrical and Electronics Engineers

IETF Internet Engineering Task Force

IP Internet Protocol

ipconfig IP Configuration (a command line used in Command Prompt)

ISP Internet Service Control

LAN Local Area Network

MAC Media Access Control

NIC Network Interface Card

OSI Open Systems Interconnection

PDU Protocol Data Unit

ping Packet Internet Groper

PLDT Philippine Long Distance Telephone (a company)

POP Point of Presence (in Internet); Post Office Protocol (in e-mail)

QoS Quality of Service

RFC Request for Comments

RJ Registered Jack

SMTP Simple Mail Transfer Protocol

TCP Transmission Control Protocol

UTP Unshielded Twisted Pair (a cable)

VECO Visayan Electric Company (only in the Philippines)

WAN Wireless Area Network

1
2
3
4

Pretest and Post-test Items

Through what process does UTP cable help to avoid crosstalk?


a. grounding the endpoints
c. twisting of pairs
b. cladding in cable
d. shielding of cable
Select the network topologies.
a. ring, mesh, star, moon
b. LAN, MAN, WAN

c. bus, star, point-to-point, mesh


d. star, bus, car, mesh

The protocol that automates the assignment of IP addresses, subnet masks, gateways, and other IP
networking parameters.
a. FTP
c. HTTP
b. DHCP
d. TCP/IP
_____ are inbound mail delivery protocols and are typical client or server protocols. They deliver
email from the email server to the client.

98

Computer Networks and Data Communications: Part 1


a. POP
b. SMTP

5
6
7
8
9
10
11
12
13
14
15

c. HTTP
d. TCP/IP

_____ is a utility for testing IP connectivity between two hosts.


a. ping
c. ipconfig
b. PDU
d. CMD
When two devices transmit at the same time on the same channel, this is called _____.
a. error
c. boom
b. collision
d. medium problem
Physical addresses are codes placed on the LAN adapter by the manufacturer, it is also known as
_____.
a. frame
c. application
b. MAC address
d. PDU
To ensure that the content of the received frame at the destination matches that of the frame that
left the source node, a transmitting node creates a logical summary of the contents of the frame.
This is known as _____.
a. hamming code
c. CRC
b. error-trapping
d. parity system
_____ is a message sent from one host to all other hosts on the network.
a. Information
c. Data
b. Packet
d. Broadcast
The following are examples of ISPs except _____.
a. Sky Cable
c. Smart Broadband, Inc.
b. VECO
d. PLDT
Another name for LAN adapter.
a. interface
b. NIC

c. wireless, copper, fiber optic cable


d. physical port

_____ is the transfer rate of actual usable data bits.


a. Throughput
c. Bandwidth
b. Goodput
d. Frequency
_____ is the capacity of a medium to carry data in a given amount of time.
a. Frequency
c. Download time
b. Throughput
d. Bandwidth
A type of network used to connect networking devices that are in a very close geographic area such
as floor of a building itself, or a campus environment.
a. server
c. LAN
b. WAN
d. MAN
UTP (cable) stands for _____.

Computer Networks and Data Communications: Part 1


a. Unshielded Twisted Pair
b. None of the above

16
17
18
19
20
21
22
23
24
25

99

c. Unprotected Twisted Pair


d. Unprotect True Pair

The Ethernet standard 1000BASE-T has a maximum segment length of _____.


a. 2 km
c. 100 m
b. 80 km
d. 25 m
_____ is the router interface that is connected to the same network as the host, and is needed to
send a packet out of a local network.
a. Subnet
c. Bridge
b. MAC address
d. Default gateway
Which of the following are true about IP?
a. IP stands for International Protocol
b. IP operates at OSI Layer 2.

c. IP analyzes presentation layer data.


d. IP is the most common network layer
protocol.

A network of interconnected computers belonging to one organization but insulation from the
Internet.
a. Ethernet
c. MAN
b. Intranet
d. LAN
This cable has a single coated copper wire center and an outer metal mesh that acts as both a
grounding circuit and an electromagnetic shield to reduce interference.
a. AWG 22 copper cable
c. coaxial cable
b. UTP cable
d. RJ-46
A network protocol that is likely to replace IPv4.
a. Novell IPX
c. IPv5
b. IPv6
d. AppleTalk
To prevent complete chaos on the media, a _____ process is used to first detect whether the media
is carrying a signal.
a. IETF
c. TCP
b. RFC
d. CSMA
The IEEE standard for wireless LAN.
a. IEEE 802.9
b. IEEE 802.11

c. IEEE 802.5
d. IEEE 802.3

The most common UTP cable connector in LAN devices.


a. RJ-26
c. RJ-48
b. RJ-42
d. RJ-45
In a ring topology, a controlled MAC technique is used called _____.
a. segmentation
c. source-destination method
b. framing
d. token passing

100

26
27
28
29
30
31
32
33
34
35
36
37

Computer Networks and Data Communications: Part 1


T_____ is the actual transfer rate of data over the medium in a period of time.
a. Download time
c. Frequency
b. Throughput
d. Bandwidth
IPv4 address is composed of how many bits?
a. 16
b. 32

c. 64
d. 128

The number of bits in the IPv6 address.


a. 16
b. 32

c. 64
d. 128

_____ is an application layer protocol developed to allow file transfers between a client and a server.
a. POP
c. TCP/IP
b. FTP
d. SMTP
The default port number assigned to web-serving applications (using HTTP).
a. 20
c. 80
b. 110
d. 25
The transport layer has to break the data into smaller pieces called _____.
a. headers
c. PDU
b. segments
d. information
Who translate an entered web address name on a web browser into a numeric IP address?
a. IP
c. ISP
b. browser
d. DNS
The process of adding control information as data passes through the layered OSI model is called
_____.
a. PDU
c. header
b. encapsulation
d. TCP/IP
A type of cable which uses light pulses conducted through special glass conductors to carry data.
a. coaxial cable
c. UTP cable
b. fiber optic cable
d. crystal ball
Which layers of the OSL model are combined into other layers of the TCP/IP model? (Choose all
that apply.) A. Network; B. Presentation; C. Internet; D. Data link; E. Application; F. Physical;
G. Session; H. Network access; I. Transport
a. B, D, F, G
c. C, D, G, I
b. C, D, H, I
d. A, D, H, I
A protocol that governs the way that a web server and a web client interact.
a. Network Access Protocols
c. HTTP
b. Transport Protocol
d. IP
Which term describes a specific set of rules that determine the formatting of messages and the process

Computer Networks and Data Communications: Part 1


of encapsulation used to forward data?
a. QoS
b. segmentation

38
39
40

101

c. protocol
d. multiplexing

Which OSI layer is associated with IP addressing?


a. transport
c. data link
b. physical
d. network
A LAN technology specified in IEEE 802.3.
a. router
b. Ethernet

c. packet-switching
d. wireless LAN

An end device or equipment which is either the source or destination of information on a network.
a. terminal
c. host
b. computer
d. workstation

102

1
2
3
4
5
6
7
8
9
10
11
12

102

Answers

(c) The UTP cable shall be twisted the


pairs very consistently to avoid crosstalk.
(c) Common network topologies are bus,
star, point-to-point, and mesh.
(b) DHCP automates the assignment of IP
address, subnet masks, gateways, and other
IP networking parameters.
(a) POP are inbound mail delivery
protocols and are typical client or server
protocols.
(a) Ping is a utility for testing IP
connectivity between two hosts.
(b) When two devices transmit at the same
time on the same channel, this is called
collision.
(b) MAC addresses (or physical addresses)
are codes placed on the LAN adapter by
the manufacturer.
(c) To ensure that the content of the
received frame at the destination matches
that of the frame that left the source node, a
transmitting node creates a logical summary
of the contents of the frame. This is known
as CRC.
(d) Broadcast is a message sent from one
host to all other hosts on the network.
(b) VECO does not service ISP but an
electrical utility in the Central Visayan
region of the Philippines.
(b) NIC is another name for LAN adapter.
(b) Goodput is the transfer rate of actual
usable data bits.

13
14
15
16
17
18
19
20
21
22
23

(d) Bandwidth is the capacity of a medium


to carry data in a given amount of time.
(c) LAN used to connect networking
devices that are in a very close geographic
srea such as floor of a building, a building
itself, or a campus enviroment.
(a) UTP stands for Unshielded Twisted
Pair.
(c) The Ethernet standard 1000BASE-T has
a maximum segment length of 100 m.
(d) Default gateway is the router interface
that is connected to the same network as
the host, and is needed to send a packet out
of a local network.
(d) The statement is true that IP is the
most common network layer protocol.
(b) Intranet is a network of interconnected
computers belonging to one organization
but insulation from the Internet.
(c) The coaxial cable has a single coated
copper wire center and an outer metal
mesh that acts as both a grounding circuit
and an electromagnetic shield to reduce
interference.
(d) IPv6 (which is currently in use in some
areas) is likely to replace IPv4 (which is
most widely used IP, and basic protocol of
the Internet).
(d) To prevent complete chaos on the
media, a CSMA process is used to first
detect whether the media is carrying a
signal.
(b) The IEEE standard for wireless LAN is
IEEE 802.11.

Computer Networks and Data Communications: Part 1

24
25
26
27
28
29
30
31
32
33
34
35

(d) RJ-45 is the most common UTP cable


connector in LAN devices.

103

side by side for comparison.


OSI Model

(d) In a ring topology, a controlled MAC


technique is used called token passing.

TCP/IP Model

Application
Presentation

Application

Session

(b) Throughput is the actual transfer rate


of data over the medium in a period of
time.

Transport

Transport

Network

Internet

(b) IPv4 address has 32 bits.

Data Link
Physical

(d) IPv6 address has 128 bits.

Fig.I1
When juxtaposed, you can see that the
functions of the application, presentation,
and session layers of the OSI model are
combined into one application layer in the
TCP/IP model.

(b) FTP is an application layer protocol


developed to allow file transfers between a
client and a server.
(c) The port number 80 (or TCP port 80)
is a default port assigned to web-serving
applications.

The bulk of networking functions reside


at the transport and the network layers, so
they remain individual layers. TCP operates
at the transport layer, and IP operates at the
Internet layer.

(b) The transport layer has to break the data


into smaller pieces called segments.

The data link and physical layers of the OSI


model combine to make the network access
layer of the TCP/IP model.

(d) DNS translates an entered web address


name on a web browser into a numeric IP
address.

Source: Network Fundamentals, CCNA


Exploration Companion Guide: Dye, M.A;
McDonald, R.; Rufi, A.W.

(b) The process of adding control


information as data passes through the
layered OSI model is called encapsulation.

To solve the problem, identify the


combined layers from OSI model into
TCP/IP model. Therefore, based on the
choices, the layers are presentation,
session, data link, and physical.

(b) The fiber optic cable uses conducting


light pulses through special glass conductors
to carry data.
(a) Hint: Layers of OSI Model and TCP/
IP Model
The TCP/IP model evolved faster than
the OSI model and is now more practical
in describing network communication
functions. The OSI model described in
detail functions that occur at the upper
layers on the hosts, while networking is
largely a function of the lower layers. Fig.I1
shows the two models

Network Access

36
37
38

(c) HTTP governs the way that a web


server and a web client interact.
(c) The protocol describes a specific set
of rules that determine the formatting of
messages and the process of encapsulation
used to forward data.
(d) The network layer is associated with IP
addressing.

104

39
40

Computer Networks and Data Communications: Part 1


(b) Ethernet is a LAN technology specified
in IEEE 802.3.
(c) The host is an end device or equipment
which is either the source or destination of
information on a network.
Most definitions are stated in the reference:
Network Fundamentals, CCNA Exploration
Companion Guide: Dye, M.A; McDonald,
R.; Rufi, A.W.

1
2
3
4
5
6
7
8
9
10
11

105

Wave Propagation

Pretest and Post-test Items

What is not a criterion for circular polarization?


c. E must have two orthogonal components.
a. E must have no component along the
direction of propagation.
d. Components of E must be 90 out of phase.
b. Components of E must be equal amplitudes.
With tan 1, what is the medium of propagation?
a. good conductor
c. lossy dielectric
b. lossless dielectric
d. free space
With tan 1, what is the medium of propagation?
a. good conductor
c. lossless dielectric
b. lossy dielectric
d. free space
With tan 1, what is the medium of propagation?
a. good conductor
c. lossless dielectric
b. lossy dielectric
d. free space
If , then the medium for the wave propagation is _____.
a. good conductor
c. lossy dielectric
b. lossless dielectric
d. free space
If , then the medium for the wave propagation is _____.
a. good conductor
c. lossy dielectric
b. lossless dielectric
d. free space
With = 0, = 0r, and = 0 r, what is the medium of propagation?
a. good conductor
c. lossy dielectric
b. lossless dielectric
d. free space
The resistance for a unit length with cross-sectional area of 1 is _____.
a. intrinsic impedance
c. ac resistance
b. dc resistance
d. surface resistance
The real part of for a good conductor is known as _____.
a. intrinsic impedance
c. ac resistance
b. dc resistance
d. surface resistance
In a good conductor, the fields tend to concentrate within the initial distance of from the conductor
surface.
a. Poyntings theorem
c. skin effect
b. phase matching conditions
d. loss tangent
It describes how the path of a light ray changes when it moves into a material with a different index
of refraction.

106

Wave Propagation
a. Snells law
b. Brewsters angle

12
13
14
15
16
17
18
19
20

c. Fermats principle
d. Fresnels equations

The light travels the path which takes the least time.
a. Snells law
c. Fermats principle
b. Brewsters angle
d. Fresnels equations
The equation = + j is the mathematical expression for _____.
a. Helmholtzs equation
c. skin depth
b. wave equation
d. wavenumber
The equation = 2/ is the mathematical expression for _____.
a. Helmholtzs equation
c. skin depth
b. wave equation
d. wavenumber
Given E = 10 e0.1z cos (1 000 t 0.2 z) a x V/m, what is the velocity of propagation of the wave in a
good conductor?
a. 1 000 a x m/s
c. 1 000 a x m/s
b. 159.155 a z m/s
d. 159.155 a z m/s
Given E = 10 e0.1z cos (1 000 t 0.2 z) a x V/m, what is the direction of the magnetic field intensity?
c. a y
a. a z
b. a x
d. a x
Given E = 10 e0.1z cos (1 000 t 0.2 z) a x V/m, what is the amplitude of the electric field intensity
10 meters away from the source?
a. 3.679
c. 2
b. 2
d. 3.679
Given E = 10 e0.1z cos (1 000 t 0.2 z) a x V/m, what is the phase difference between E and H in
a lossless dielectric?
a. zero
c. indeterminate
b.
d. 45
What is the polarization of E = cos a x + 0.4 sin a y?
a. left-hand circular
c. left-hand elliptical
b. right-hand circular
d. right-hand elliptical
What is the polarization of E = cos a x sin a y?
a. left-hand circular
c. left-hand elliptical
b. right-hand circular
d. right-hand elliptical

1
2
4

2
3
4
5

Answers

(a) The statement in Letter a is not a


criterion for circular polarization.
Items 2 to 4 has different inequalities
between tan and the ratio, where the
angle is specifically called Brewsters
angle (or polarization angle). Here, the
tan is used to determine, if the medium
of propagation is either a conductor or a
dielectric, by the ratio of the incident and
reflected ray of the propagated light.
(c) With tan 1, the medium of
propagation is a lossy dielectric.
(a) With tan 1, the medium of
propagation is a good conductor.
(b) With tan 1, the medium of
propagation is a lossless dielectric.
Items 5 to 7 has the same given quantities
(where = conductivity, = angular
frequency, = permittivity of a material)
showing with different inequalities. These
are originality derived from the current
density.

(a) If , then the medium for the


wave propagation is good conductor.
(c) If , then the medium for the
wave propagation is lossy dielectric.
(b) If = 0, = 0r, and = 0 r, then
the medium for the wave propagation is
lossless dielectric.
(b) The resistance for a unit length with
cross-sectional area of 1 (distance of
the conductor surface) is dc resistance.
(d) The real part of (intrinsic impendace)

for a good conductor is known as surface


resistance.

10
11
12
13
14
15

7
5
6
7
8
9

107

18
15

(c) In a good conductor, the fields tend to


concentrate within the initial distance
from the conductor surface. This
phenomenon is called skin effect.
(a) Snells law describes how the path of
a light ray changes when it moves into a
material with a different index of refraction.
(b) Fermats principle simply states that
the light travels the path which takes the
least time.
(Not available) The equation = + j
is the mathematical expression for
propagation costant.
(d) The equation = 2/ is the
mathematical expression for wavenumber.
Given E = 10 e0.1z cos (1 000 t 0.2 z) a x
V/m comes from the format of E(z, t) =
E0 ez cos (t z) a x; in polarization, the
wave of the electric field travels with time
along z-axis, and the amplitude heads at
x-axis.
(b) Hint: Velocity of Propagation
vp = /

Eq.J1

The velocity of a wave is the rate at which


the phase of the wave propagates in space.
Eq.J1 is given in terms of the waves
angular frequency and wavenumber.
Given: = 1 000 rad/s (radian per second)
Since = 2/ and the wavelength is not
given, assume that the wave is one complete
cycle that exists in one meter, that is = 1 m;
so = 2 rad/m.
Note that = 2f. Then,

108

Wave Propagation
Given: E = cos a x + 0.4 sin a y

= 1 000 rad/s = 2f
f 159.155 Hz

We immediately say that the given electric


field is elliptical polarization because it has
a major and minor axis, 1 and 0.4.

For vp,

2(159.155) rad/s
vp = = 159.155 m/s
2 rad/m

We can use the technique called the axial


ratio (AR) to determine either right-hand
or left-hand elliptical polarization. It is ratio
of the major to the minor axis (Eq.J2).
major axis
AR =
Eq.J2
minor axis

The velocity of propagation can be a vector


quantity. Knowing that the given electric
field E indicates with the unit vector a x, the
direction of travel is along z-axis and the
rotation of the wave is counterclockwise
(by using the right-hand rule) as which
the phase goes to the +z-axis; therefore:
v p = +159.155 a z m/s
Do not mind the good conductor (or
whatever type of medium) in this case
because in polarization of electromagnetic
(EM) waves, this affects only in the
magnetic field based on the principle:
electric field E leads magnetic field H.

16

(c) Hint: Right-Hand Rule


The direction of propagation of EM waves
is given by the right-hand rule. To point
the fingers of your right hand toward the
E, curl your fingers towards the H, and
the thumb will the point the direction of
propagation.
We represent the vector of electric field along
x-axis, the vector of magnetic field along
y-axis, and the direction of propagation
along z-axis. Since the rotation of the wave
is counterclockwise in Item 15, we conclude
that the E is positive unit vector a x while
the H is positive unit vector a y.

17
18
19

(a) Let cos (1 000 t 0.2 z) = 1 because


the amplitude of electric field E heads at
x-axis. With z = 10 m, then,
E = 10 e0.1(10) = 3.679 V/m
(a) In a lossless dielectric, the phase
difference between E and H is zero. So E
and H are in time phase.
(c) Hint: Axial Ratio

Then,

1
= +2.5
0.4

The AR is positive, so this is a left-hand


elliptical polarization.

20

(a) Hint: Determining the Angular


Frequency using Tangent
Given: E = cos a x sin a y
The given electric field is circular
polarization where the components of
electric field, |Ex| = |Ey| = 1, and AR = 1.
Since the components are perpendicular,
the tangent is given by Eq.J3.
E
Eq.J3
tan1 y
Ex
With trigonometric identity, we obtain:
sin
tan1 = tan1 (tan ) =
cos
Be careful that here represents the
angular frequency. Based on the solution,
the angular frequency is negative; so the
wave is left-hand circular polarization
because it rotates counterclockwise in the
sense when it is viewed along the axis of
propagation.

K
1
2
3
4
5
6
7
8
9
10
11

Analog Communications and


Broadcasting

109

Pretest and Post-test Items

What is not a property of FM signals?


a. nonlinearity of modulation process
b. None of these

c. regularity of zero crossings


d. constancy of transmitted power

Frequency limits of Channel 29 in the TV broadcast band.


a. 560 MHz to 566 MHz
c. 554 MHz to 560 MHz
b. 572 MHz to 578 MHz
d. 556 MHz to 572 MHz
(t + T) = g(t), and (t) = g(t T)
a. None of these
b. time inversion

c. time scaling
d. time shifting

Radio emission code designation of ITU for frequency-shift keyed data.


a. F2D
c. G7E
b. A3E
d. J3E
Carrier frequency of Channel 43 in the AM broadcast channel.
a. 927 kHz
c. 936 kHz
b. 918 kHz
d. 909 kHz
A signal known only in terms of probabilistic description is _____.
a. deterministic signal
c. power signal
b. random signal
d. energy signal
In a standard single-tone AM signal with 100% modulation at 1 W total output power, how much
power is in the lowest frequency generated?
a. 125 mW
c. 166.67 mW
b. 62.5 mW
d. 55.56 mW
(1/2) = g(t), and (t) = g(2t)
a. time shifting
b. None of these

c. time inversion
d. time scaling

For efficient radiation of EM wave, the antenna must be at least _____ of the wavelength of the
highest signal transmitted.
a. one-twentieth
c. one-tenth
b. one-hundredth
d. one-half
In direct FM, fi(t) = fc + km(t) and k is known as _____.
a. arbitrary constant
c. frequency sensitivity
b. unknown constant
d. phase sensitivity
In a standard single-tone AM signal with 50% modulation at 1 W total output power, how much
power is in the highest frequency generated?

110

Analog Communications and Broadcasting


a. 166.67 mW
b. 55.56 mW

12
13
14
15
16
17
18
19
20

c. 62.5 mW
d. 125 mW

Radio emission code designation of ITU for single-sideband voice.


a. A3E
c. J3E
b. G7E
d. F2D
What are the information given during station identification announcement of a TV broadcast station?
a. channel and location
c. call letters and channel
b. call letters, channel and location
d. call letters and location
The peak difference between instantaneous of the modulated wave and the carrier signal.
a. frequency deviation
c. bandwidth
b. maximum frequency
d. sideband
Which is not part of a TV broadcast band?
a. 54 MHz to 72 MHz
b. 174 MHz to 216 MHz

c. 76 MHz to 88 MHz
d. 108 MHz to 170 MHz

To achieve pre-emphasis in FM, the signal frequencies above cut-off _____.


a. increase in level by 6 dB per decade
c. increase in level by 6 dB per octave
b. increase in level by 3 dB per decade
d. increase in level by 3 dB per octave
Which component of radiation is taken as reference in circular or elliptical polarizaed antenna for
FM broadcasting?
a. elliptical
c. horizontal
b. circular
d. vertical
Carrier frequency of Channel 228 in the FM broadcast band.
a. 93.5 MHz
c. 93.9 MHz
b. 94.1 MHz
d. 93.7 MHz
The band of frequencies from 23 kHz to 99 kHz containing sound subcarriers and their associated
sidebands.
a. sound sub-channel
c. sub-channel
b. crosstalk
d. stereophonic sound sub-channel
(t) = g(t) and (t) = g(t)
a. time shifting
b. None of these

c. time inversion
d. time scaling

K
1
2

111

Answers

(c) The regularity of zero crossings is not


a property of FM (frequency modulation)
signals.

words, the frequency band of Channel 14


(VHF) to the UHF is straightly designated
for TV broadcast band.

Instead, this allows in PM (phase


modulation).

Let the quantities a1 = 470 MHz, n = (29


14) + 1 = 16 (meaning Channel 29 is at the
16th term), d = 6 MHz. Then,
470 106 + (16 1)(6 106) = 560 MHz

(a) Hint: nth Term of an Arithmetic


Sequence (See Eq.A2 in Mathematics)
This is an alternative solution to determine
the lower-side frequency band of a certain
channel.
The TV (television) broadcast band is
categorized into two frequency spectra, the
VHF (very high frequency) and UHF (ultra
high frequency) TV channels. The VHF
ranges about 30 MHz to 300 MHz while
the UHF ranges about 300 MHz to 3 GHz.
We shall follow the TV standards in NTSC
(National TV System Committee). Each
frequency band of a TV channel allocates
6 MHz. The first TV channel, that is
Channel 2, starts at 54 MHz within the limit
at 60 MHz. The frequency band of the next
channel is 60 MHz to 66 MHz, and so on.

The lower-side frequency band of Channel


29 is 560 MHz and its upper-side frequency
band is 566 MHz.

3
4
5

Notice that the preceding frequencies


sequence arithmetically. However, be
mindful that some TV channels have
skipped frequency bands just like in
Channel 6 to 7 with the range of 82 MHz
to 88 MHz, and 174 MHz to 180 MHz,
respectively; so the sequence disconnects
there. This is because 88 MHz to 174 MHz
are used for other modulations such as FM.

(a) The radio emission code designation


of ITU (International Telecommunications
Union) for frequency-shift keyed data is
F2D.
(d) Hint: nth Term of an Arithmetic
Sequence and Median Value
The AM (amplitude modulation) broadcast
band has a frequency spectrum from
526.5 kHz to 1 705 kHz. There are more
than 100 channels with a frequency band
allocation of 9 kHz. For the first channel,
Channel 1 starts at 526.5 kHz to 535.5 kHz.
Frequency in every channel sequences
arithmetically.
Given: Channel 43
Required: carrier frequency
Using again Eq.A2, let the quantities a1 =
526.5 kHz, n = 43, d = 9 kHz. Then,
526.5 103 + (43 1)(9 103) = 904.5 kHz

Given: Channel 29
Required: frequency band

The carrier frequency is simply the median


between the lower-side and upper-side
frequency band. So,
(904.5 103 + 913.5 103)/2 = 909 kHz

Recalling Eq.A2 is:


an = a1 + (n 1)d
The good arithmetic sequence here starts at
the frequency band of Channel 14, that is
470 MHz, and this will be used as the first
term a1. It is so-called good because the
sequence does not break off. In other

(d) The mathematical functions, (t + T) =


g(t), and (t) = g(t T) describe a time
shifting.

(b) A signal known only in terms of


probabilistic description is random signal.

112

Analog Communications and Broadcasting


(c) Hint: Total Average Power of a SingleTone AM Signal
m2
Pt = Pc 1 +
2

Eq.K1

where Pt = the total average power, Pc =


power of the carrier (frequency), m =
modulation index.
The modulated AM signal for a singletone is shown in Fig.K1 which is basically
visualized on the frequency domain. The
three terms represent the carrier, upper
sideband (or upper-side frequency band)
and lower sideband. When the carrier is
modulated, its amplitude is higher than the
two sidebands.
carrier
lower
sideband

upper
sideband

f
Fig.K1
Given: Pt = 1 W; m = 100% or 1
Required: power of the lower sideband
Solve Pc first.
12
1 = Pc 1 +
2
Pc = 2/3 W
The Pt is the total of the Pc, power of the
lower sideband, and power of the upper
sideband. Since Pt > Pc, subtract Pc from Pt
to get the total power in the two sidebands.
Pt Pc = 1 2/3 = 1/3 W
Notice that two sidebands have equal power,
so the lower and upper sideband will be
(1/3)/2 = 166.67 mW

8
9

(d) The mathematical functions (1/2) =


g(t), and (t) = g(2t) describe a time
scaling.
(b) For efficient radiation of EM wave, the

antenna must be at least one-hundredth


of the wavelength of the highest signal
transmitted.

10
11

(c) In direct FM, fi(t) = fc + km(t) and k is


known as frequency sensitivity.
(b) Hint: Total Average Power of a SingleTone AM Signal (See Eq.K1)
Given: Pt = 1 W; m = 50% or 0.5
Require: power of the upper sideband
The method is the same as Item 7. Solving
Pc yields 8/9 W. Then Pt Pc = 1/9 W. For
the power of each sideband, it is 55.56 mW.

12
13
14
15
16
17
18

(c) The radio emission code designation of


ITU for single-sideband voice is J3E.
(b) The call letters, channel and location
are the information given during the station
identification announcement of a TV
broadcast station.
(a) Frequency deviation is the peak
difference between instantaneous of the
modulated wave and the carrier signal.
(d) The range from 108 MHz to 170 MHz
is not part of a TV broadcast band.
Due to standards, this range is used for
other modulations including FM.
(c) To achieve pre-emphasis in FM, the
signal frequencies above cut-off increase in
level by 6 dB per octave.
(d) The vertical component of radiation is
taken as reference in circular or elliptical
polarized antenna for FM broadcasting.
(a) Hint: nth Term of an Arithmetic
Sequence and Median Value
The FM broadcast band has a frequency
spectrum from 88 MHz to 108 MHz. There
are 100 channels with a frequency band
allocation of 200 kHz. For the first channel,

Analog Communications and Broadcasting


Channel 201 starts at 88 kHz to 88.2 kHz.
Frequency in every channel sequences
arithmetically.
Given: Channel 228
Required: carrier frequency
Using again Eq.A2, let the quantities a1 =
88 MHz, n = 28 (28th term in Channel 228),
d = 200 kHz. Then,
88 106 + (28 1)(200 103) = 93.4 MHz
For the carrier frequency,
(93.4 106 + 93.6 103)/2 = 93.5 MHz

19
20

(d) Stereophonic sound sub-channel has


the band of frequencies from 23 kHz to
99 kHz containing sound subcarriers and
their associated sidebands.
(c) The mathematical functions (t) = g(t)
and (t) = g(t) describe a time inversion.

113

114

Digital and Data


Communications: Part 2

114

Abbreviations which are mentioned in the items are defined.



B and D Bearer and Data

CB radio Citizens Band radio

CSMA/CD Carrier Sense Multiple Access/Collision Detect

DCE Data Communication Equipment

DTE Data Terminal Equipment

FSK Frequency-Shift Keying

IBM International Business Machines (company)

ISDN Integrated Services for Digital Network

PCM Pulse Code Modulation

PSK Phase-Shift Keying

PSN Packet Switching Network

QAM Quadrature Amplitude Modulation

Rx/Tx Receiver/Transmitter

RS# Recommended Standard (number)

URL Uniform Resource Locator

1
2
3
4
5
6

Pretest and Post-test Items

A word in telegraphy consists of _____ characters plus a word space.


a. 5
c. 11
b. 7 and 1/2
d. 8
One character or a sequence of characters forming a part, or the whole of a message with a specific
meaning.
a. identifier
c. code
b. signs
d. call sign
The standard that specifies a balanced interface cable that will operate bit rates up to 10 Mbps with
a span distance up to 1 200 m.
a. RS-449A
c. RS-423A
b. RS-422A
d. RS-550A
What is one principal difference between synchronous and asynchronous transmission?
a. The bandwidth required is different.
c. The clocking is mixed in the data in
b. The clocking is derived from the data in
asynchronous.
synchronous transmission.
d. The pulse heights are different.
What is the transmission rate of a system for transmitting the output of a microcomputer to a line
printer operating at a speed of 30 lines/minute? Assume that the line printer has 8 bits of data per
character and prints out 80 characters/line.
a. 400 bps
c. 320 bps
b. 800 bps
d. 640 bps
Modulation in which no signal is present between pulses.
a. QAM
c. FSK
b. PSK
d. Pulse modulation

Digital and Data Communications: Part 2

7
8
9
10
11
12
13
14
15
16
17
18

Modem is referred to as _____.


a. DTE
b. Universal Asynchronous Rx/Tx

115

c. DCE
d. Universal Synchronous Rx/Tx

A seven-bit character can represent one of _____ possibilities.


a. 64
c. 128
b. 7
d. 14
An invitation from the primary to a secondary to transmit a message.
a. reuse
c. selection
b. retransmission
d. polling
Digital modulation technique used in modes.
a. FSK
b. PSK

c. QAM
d. All of these

Transmission sent in both directions simultaneously.


a. half-duplex
c. simplex
b. duplex
d. full-duplex
_____ is a device in data transmission to interface a DTE to an analog transmission line.
a. RJ-11 plug
c. Modem
b. RS232
d. DTE
A source code whose average word length approaches the fundamental limit set by the entropy of
a discrete memoryless source.
a. source code
c. Huffman code
b. entropy code
d. prefix code
In _____ transmission, a unique SYN character is transmitted at the beginning of each message.
a. digital
c. asynchronous
b. synchronous
d. analog
For a sample rate of 40 kHz, determine the maximum analog input frequency.
a. 10 kHz
c. 30 kHz
b. 20 kHz
d. 40 kHz
Which of the following described the very early standard that defines binary digits as space per mark
line conditions and voltage levels?
a. V.1
c. V.4
b. V.2
d. V.5
All bits in a character are sent and received _____ in a serial port.
a. simultaneously
c. one at a time
b. in groups of 2 bits
d. in groups of 3 bits
Which of the following is not referred to as DTE?

116

Digital and Data Communications: Part 2


a. printers
b. modem

19
20
21
22
23
24
25
26
27
28
29

c. telephone set
d. computer

When one Boolean operation has a result which is negative of another, the two operations are said
to be _____.
a. inverse
c. supplementary
b. complementary
d. adjunct
Mode of communication system where both sides of the system wait for their turn to transmit at a time.
a. full-duplex
c. telephone
b. half-duplex
d. CB Radio
What is IBMs asynchronous data link protocol designation?
a. 83B
c. 93B
b. 9A1/9B1
d. 8A1/8B1
Asynchronous protocol is _____.
a. bit oriented
b. message oriented

c. clock oriented
d. character oriented

Ethernet is baseband system using CSMA/CD operating at _____.


a. 40 Mbps
c. 20 Mbps
b. 10 Mbps
d. 30 Mbps
Transmission system for a multidrop network.
a. CSMA/CD
b. polling

c. token passing
d. switching

What equation defines the composition of an ISDN basic access line?


a. 2B + 2D
c. B + D
b. B + 2D
d. 2B + D
_____ is a protocol used to connect the other PSN.
a. X.10
c. X.50
b. X.25
d. X.75
Highest theoretical frequency that can be processed at a sampling rate without aliasing.
a. critical frequency
c. resonant frequency
b. natural frequency
d. folding frequency
Data is transmitted directly on the transmission cable.
a. synchronous
c. broadband
b. baseband
d. asynchronous
12 voice channels are sampled at 8 000 sampling rate and encoded into 8-bit PCM word. Determine
the rate of the data stream.
a. 750 kbps
c. 640 kbps
b. 345 kbps
d. 768 kbps

Digital and Data Communications: Part 2

30
31
32
33
34
35
36
37
38
39
40

117

A device that connects two dissimilar networks and performs the protocol conversion.
a. converter
c. coupler
b. gateway
d. transformer
The total useful information processed or communicated over a specified amount of time.
a. throughput
c. bit rate
b. information technology
d. baud rate
Determine from the following the basic mode of transmission system in the public data network in
which data are transferred from the source to the network and from the network to the destination
in the frame format.
a. packet mode
c. asynchronous mode
b. voice mode
d. synchronous mode
These are used to connect non-ISDN equipment to ISDN line.
a. terminal repeaters
c. digipeaters
b. terminal adapters
d. local repeaters
One of the commonly used domain in the Internet service.
a. www
c. http
b. .com
d. Yahoo
In communications, when we call data communications, it means the transmission of _____.
a. computer data
c. voice
b. video
d. voice and video
An indication signal for a handshake from the DTE in response to an active condition request to
send a signal.
a. data ready
c. clear to send
b. receive data
d. data transmission
Main basic components of a data communication system are composed of the following.
a. transmitter, computer and modem
c. computer, modem and router
b. transmitter, channel and receiver
d. computer, modem and gateway
Refers to an address that identifies a server on the network and a particular document on the server.
a. URL
c. key transmission
b. digital emission
d. cycle transmission
Who is responsible in the correctness and accuracy of transmitted information content over a data
communication?
a. bridge
c. channel
b. modem
d. transmitter and receiver
Device that complies with the ISDN interface and can be connected directly to the digital network.
a. Modem
c. TE1 (ISDN terminal)
b. TE2 (Non-ISDN terminal)
d. TA (Terminal Adapter)

118

118

1
2
3
4
5

Answers

(b) A word in telegraphy consists of 7 and


1/2 characters plus a word space.
(c) The code consists a one character or
a sequence of characters forming a part,
or the whole of a message with a specific
meaning.
(b) RS-422A is a standard cable that
specifies a balanced interface cable that will
operate bit rates up to 10 Mbps (mega-bits
per second) with a span distance up to 1.2 km.
(b) One principal difference between
synchronous and asynchronous transmission
is that the clocking is derived from the
data in synchronous transmission.
(c) Hint: Consistency of Units
The unit of bit rate, which is the
requirement, is bit per second (bps).
Though there is a formula for bit rate,
the given in the problem is useless; so the
solution is to set the units of the given until
the unit of bit rate is attained. Conversions
might be also necessary.
Given: operating speed of the system =
30 lines/minute or 0.5 lines/second; number
of characters per line = 80 characters/line;
number of bits containing in a character =
8 bits/character
Require: bit rate (or transmission rate)
As the units are settled, the bit rate is:
(0.5 lines/second)(80 characters/line)
(8 bits/character) = 320 bps

6
7
8

Hint: Input Combinations in Binary


Recalling Eq.G1 in Digital Electronics is:
m = 2n
With n = 7 bits, then,
27 = 128

9
10
11
12
13
14
15

(d) Pulse modulation has no presence of


signal between pulses.

(d) FSK, PSK, (also ASK, amplitude-shift


keying) and QAM are digital modulation
technique used in modes.
(d) Full-duplex transmission sent in both
directions simultaneously.
(c) Modem is a device in data transmission
to interface a DTE to an analog
transmission line.
(c) Huffman code is a source code whose
average word length approaches the
fundamental limit set by the entropy of a
discrete memoryless source.
(b) In synchronous transmission, a
unique SYN character is transmitted at the
beginning of each message.
(b) Hint: Nyquist frequency
fN 2fs
where fN = maximum (or Nyquist)
frequency, fs = sampling (or input)
frequency. Nyquist frequency (known as
a folding frequency) is twice the highest
frequency present in a signal. (See
definition in Item 27.)
Given: fN = 40 kHz
Require: fs

(c) Modem is referred to as DCE.


(c) A seven-bit character can represent one
of 128 possibilities.

(d) Polling is an invitation from the primary


to a secondary to transmit a message.

40 103/2 = 20 kHz

16

(a) V.1 describes the very early standard


that defines binary digits as space per mark

Digital and Data Communications: Part 2


line conditions and voltage levels.

17
18
19
20
21
22
23
24
25
26
27
28
29

(c) All bits in a character are sent and


received one at a time in a serial port.
(b) Modem is not referred to as DTE.
(b) When one Boolean operation has a
result which is negative of another, the two
operations are said to be complementary.
(b) Half-duplex is a mode of communication
system where both sides of the system wait
for their turn to transmit at a time.
(a) IBMs asynchronous data link protocol
designation is 83B.
(d) Asynchronous protocol is character
oriented.
(b) Ethernet is baseband system using
CSMA/CD operating at 10 Mbps.
(b) Polling uses transmission system for a
multidrop network.
(d) The equation 2B + D defines the
composition of an ISDN basic access line.
(b) X.25 is a protocol used to connect the
other PSN.
(d) The highest theoretical frequency that
can be processed at a sampling rate without
aliasing is called folding frequency.
(b) The transmitted data directly on the
transmission cable is called baseband.
(d) Consistency of Units
Given: number of (voice) channels =
12 channels; sampling rate = 8 000 samples/
second; number of bits containing in a
channel = 8 bits/channel
Require: bit rate (or sampling rate)

119
12 channels(8 000 samples/second)
(8 bits/channel) = 768 kbps or
768 000 samples/second

30
31
32
33
34
35
36
37
38
39
40

(b) The gateway is a device that connects


two dissimilar networks and performs the
protocol conversion.
(a) Throughput is the total useful
information processed or communicated
over a specified amount of time.
(a) That is a packet mode.
(b) The terminal adapters are used to
connect non-ISDN equipment to ISDN line.
(d) Yahoo is one of the commonly used
domain in the Internet service.
(a) In communication, when we call data
communications, it means the transmission
of computer data.
(b) The receive data indicates a signal for a
handshake from the DTE in response to an
active condition request to send a signal.
(c) The main basic components of a data
communication system are composed of
transmitter, channel and receiver.
(a) URL refers to an address that identifies
a server on the network and a particular
document on the server.
(d) The transmitter and receiver are
responsible in the correctness and accuracy
of transmitted information content over a
data communication.
(c) TE1 (known as ISDN terminal) is
a device that complies with the ISDN
network interface and can be connected
directly to the digital network.

120

Das könnte Ihnen auch gefallen